Respiratory System/Lungs/Chest Wall Flashcards

1
Q

A 77-year-old man presents with shortness of breath and right-sided chest discomfort. A chest X- ray shows a right-sided pleural effusion. A thoracocentesis is performed and the pleural fluid sent to laboratory for evaluation. The results are as follows:
Pleural fluid protein to serum protein ratio: 0.7
Pleural fluid LDH to serum LDH:0.8
Glucose of 45mg/dl

Which one of the following is the most likely underlying cause of for this pleural effusion?
A. Congestive heart failure.
B. Hepatic cirrhosis.
C. Autoimmune hemolytic anemia.
D. Malignancy.
E. Nephrosis.

A

D. Malignancy.

Pleural effusion is accumulation of fluid in the pleural cavity. It can be transudative, exudative, or hemorrhagic.

Transudate
Transudate effusionsare caused by either increased hydrostatic pressure e.g. congestive heart failure, or decreased oncotic pressure, usually due to decreased serum albumin level, such as in chronic liver disease and cirrhosis or nephrosis in which protein is lost through the urine. Because of systemic involvement, transudate pleural effusion is often bilateral and equal in both lungs.

Exudate
Exudate effusions are caused by a local pathology such as pneumonia, tuberculosis, or
malignancies. This type of effusion is usually unilateral and needs more investigation.

Hemorrhagic
Hemorrhagic pleural effusion is seen in malignancies, trauma or pulmonary effusion.

The following table summarizes the most common causes of each type:

See table below.

Pulmonary embolism (PE) can cause both transudative and exudative types. Always consider PE if the transudate effusion cannot be attributed to any other underlying cause.

Exudateversus transudate pleural effusion:
After thoracocentesis, the effusion fluid albumin and lactated dehydrogenase (LDH) should be
measured, as well as those of the serum at the same time. The effusion is exudative if there is any of the following:
- Effusion fluid albumin to serum albumin ration>0.5
- Effusion fluid LDH to serum LDH ration>0.6
- Effusion fluid LDH>200 IU/ml

If none of the above criteria are met, the effusion is transudative.

A low pleural glucose concentration (1.6-2.8 mmol/L or 30-50 mg/dL) is seen in the following
conditions:
- Malignant effusion
- Empyema
- Tuberculosis pleuritis
- Esophageal rupture
- Lupus pleuritis

A very low glucose concentration (<1.6 mmol/L) narrows down the possibilities to rheumatoid
pleurisy or empyema.

A normal pleural fluid has a pH of 7.6 – 7.64.
Pleural fluid pH highly correlated with pleural fluid glucose levels. A pleural fluid pH of less than 7.30 with a normal arterial blood pH level is caused by the same diagnoses for low pleural fluid glucose. However, for parapneumonic effusiona low pleural fluid pH level is more predictive of complicated effusion (that requires drainage) than is a low pleural fluid glucose level.

This patient has exudative pleural effusion and of the given options only malignancies (lung, breast, lymphoma) can be the underlying cause.

References
* http://emedicine.medscape.com/article/299959-worku
* http://emedicine.medscape.com/article/299959-worku

How well did you know this?
1
Not at all
2
3
4
5
Perfectly
2
Q

A 57-year-old man, who has been a smoker for most of his adult life, presents to your clinic complaining of coughing up blood after he wakes up in the morning. Which one of the following is the most important condition to exclude?
A. Bronchiectasis.
B. Leukemia.
C. Laryngeal cancer.
D. Bronchogenic carcinoma.
E. Pulmonary embolus.

A

D. Bronchogenic carcinoma.

Coughing up blood indicates a pulmonary pathology, requiring a thorough investigation. Pulmonary tuberculosis and bronchogenic carcinoma should always be considered. Although bronchiectasis, COPD, bronchitis, etc can all cause cough as the most common cause of hemoptysis, since bronchogenic carcinoma has a significant aggressive course than other mentioned conditions and should be considered and excluded first.

How well did you know this?
1
Not at all
2
3
4
5
Perfectly
3
Q

Which one of the following could trigger an asthma attack?
A. Sulphites.
B. Benzyl benzoate.
C. Lecithin.
D. Terbutaline.
E. Ephedrine.

A

A. Sulphites.

The most common adverse reaction with exposure to sulfites is asthma like symptoms, estimated to occur in 5 to 10% of patients with asthma. These symptoms are more likely to occur in patients with severe asthma, or those in whom asthma is poorly controlled. Reactions can vary from mild to life-threatening.

Sulfites are used to preserve many foods and beverages. The addition of sulfites to some foods like beer and wine is permitted in most countries. However, it is illegal to add these to foods like fresh salads or fruit salads, or to meats like mincemeat. Unfortunately, these can be added from time to time illegally.

The following is a list of the most common sources of accidental exposure to sulfites.
- Drinks: cordials and some fruit juices as well as beer and wine, occasionally soft drinks and
instant tea.
- Other liquids: commercial preparations of lemon and lime juice, vinegar, grape juice.
- Commercial foods: dry potatoes, gravy and sauces and fruit toppings, maraschino cherries,
pickled onions, Maple syrup, jams, jellies, some biscuits and bread, or pie or pizza dough.
- Fruit: dried apricots, and sometimes grapes will be transported with sachets of the sulfite
containing preservative.
- Salads and fruit salads: sometimes restaurant salads and fruit salads will have sulfites added
to preserve their color.
- Crustaceans: sulphur powder is sometimes added over the top of crustaceans to stop them
discoloring.
- Meat: sulfites are sometimes illegally added to mincemeat or sausage meat.
- Other foods: gelatin, coconut.

References
* http://www.ncbi.nlm.nih.gov/pmc/articles/PMC401744
* http://www.asthmaaustralia.org.au/onAIR/Food_addit
* http://www.allergy.org.au/patients/product-allergy

How well did you know this?
1
Not at all
2
3
4
5
Perfectly
4
Q

A mother brings her 3-year-old boy to your practice with episodic asthma and persistent interval symptoms of nocturnal cough and exercise-induced wheezing. You plan to start her on fluticasone.
Which one of the following delivery methods is most appropriate for this age group?
A. Oral suspension.
B. Metered dose inhaler (MDI).
C. MDI and spacer device with mask.
D. Nebulizer and pump.
E. Breath-actuated inhaler (Accuhaler®).

A

C. MDI and spacer device with mask.

(Option A) Fluticasone is not available in form oral suspension or solution. Moreover, systemic use of corticosteroids unnecessarily poses the child at adverse effects assoiciated with systemic corticosteroids.

(Option B) Metered dose inhalers are not an effective delivery mode in this age group, as it needs breath-holding.

(Option C) MDI and spacer device with mask is the preferred mode of delivery for inhalers in young children unable to efficiently hold their breath.

(Option D) Nebulizers are also effectivebut are more expensive and difficult to use compared with spacers with face mask. They should be considered if the patient cannot cope with spacer and face mask. They are the second best option.

(Option E) Breath-actuated inhalers are inhalers that automatically release a spray of medication when the person begins to inhale. Like MDIs this mode of delivery is difficult to use in young children.

http://www.asthmawa.org.au/PageFiles/181/AA-Medica

How well did you know this?
1
Not at all
2
3
4
5
Perfectly
4
Q

Which one of the following is never indicated in acute management of an asthma attack in
children?
A. Steroids.
B. Salbutamol.
C. Ipratropium bromide.
D. Sodium cromoglycate.
E. Adrenaline.

A

D. Sodium cromoglycate.

Sodium cromoglycate is indicated in the prophylactic treatment of asthma, including prevention of exercise-induced bronchospasm. It is not useful in management of an acute asthma attack.

The management of acute asthma depends on the severity of the symptoms:
- Salbutamol (short-acting β2 agonist) is the best initial treatment in all cases of acute asthma.
- Systemic steroids are useful in moderate to severe attacks adn should be continued for 3 days and stopped without tapering.
- Ipratropium bromide (an anticholinergic agent) can also be helpful in management of moderate to severe asthma attack if maximum dose of short-acting β2 agonists fail to break the attack.
- Adrenaline can be used in severe asthma attack where there is imminent cardiorespiratory arrest or when there is anaphylactic reaction.

  • Therapeutic Guidelines – Respiratory; available from http://tg.org.au
How well did you know this?
1
Not at all
2
3
4
5
Perfectly
5
Q

A 5-year-old child is brought to your clinic with complaint of cough. His mother explains that the coughs started 2 months ago, and sometimes wakes him up at night. He also has them when he plays with other children or runs and sometimes is associated with small amount of clear phlegm. On history, you realize that he had flexural eczema when he was 2years old. There child is afebrile. On chest examination, no wheeze is heard. The rest of the physical exam is inconclusive. Which one of the following is the most likely diagnosis?

A. Epiglottitis.
B. Chronic sinusitis.
C. Asthma.
D. Gastro-esophageal reflux disease.
E. Whooping cough.

A

C. Asthma.

The age of the child, history of atopy (eczema) and chronic intermittent cough favor the diagnosis of asthma. Asthma is common inthis age group and is usually underdiagnosed. In atopic asthma, the child usually has the history of other atopies such as allergic rhinitis, eczema, etc.

(Option A) Epiglotitis has more pronounced and acute presentation with fever and cough, respiratory distress and drooling. The child is usually very ill.

(Option B) Chronic sinusitis may present with nocturnal cough caused by post nasal drip, but exertional cough is not usually a feature. Besides, the discharges tend to be more thick and purulent. There is also history of facial tenderness or fullness and recurrent upper respiratory infections.

(Option D) Gastro-esophageal reflux is rare in this age group. If present, it may causechronic calf and mimic cough-variant asthma especially at night, but not exertional cough. Moreover, absence of other features such as heart burn makes this diagnosis far less unlikely.

(Option E) Whooping cough (pertussis) has a different presentation with paroxysms of cough and the terminal inspiratory whoop.

  • http://emedicine.medscape.com/article/296301-clini
How well did you know this?
1
Not at all
2
3
4
5
Perfectly
6
Q

A 4-year-old boy, with background history of asthma, presents with a severe attack of asthma following an upper respiratory infection. On physical examination, he appears distressed with a pulse rate of 130 bpm, respiratory rate of 48 breaths per minute and temperature37.1°C. Bedside pulseoxymetry shows an oxygen saturation of 85% while on room air. Which one of the following is the most appropriate initial step in management?

A. Give salbutamol inhalation and recommend salbutamol inhaler4-hourlyat homeand review in 24 hours.
B. Start 4-hourly salbutamol and ipratropium bromide inhalersand oral prednisolone, and review in 24 hours.
C. Start inhaled salbutamol and oral amoxicillin 4-hourly, and review in 24 hours.
D. Give oxygen and continuous salbutamol inhalations and transfer to the hospital by ambulance.
E. Refer for admission to the hospital for salbutamol and ipratropium inhalations.

A

D. Give oxygen and continuous salbutamol inhalations and transfer to the hospital by ambulance.

Initial assessment of the severity of an asthma attack in children is according to the following table:

See table below.

With an oxygen saturation of less than 90%, this child has a severe asthma attack and should be transferred to the hospital for close monitoring and treatment; however, initial treatment with oxygen and short-acting β2 agonists (e.g. salbutamol) should be started without delay and maintained on the way to the hospital.

Options suggesting outpatient management are incorrect. Transferring to the hospital without oxygen supplementation and bronchodilators is inappropriate, because this boy is hypoxic and at increased risk of respiratory failure.

  • Therapeutic Guidelines – Respiratory; available from http://tg.org.au
  • http://www.asthmahandbook.org.au/uploads/Australia
How well did you know this?
1
Not at all
2
3
4
5
Perfectly
7
Q

A 25-year-old woman presents to the emergency department with exacerbation of asthma. Which one of the following is the most accurate tool to assess the severity of the attack?

A. Oxygen pressure of arterial blood (PaO2).
B. Loudness of the wheeze.
C. Severity of the dyspnoea.
D. Forced expiratory volume in one second (FEV1).
E. Pulse rate.

A

D. Forced expiratory volume in one second (FEV1).

Option A: Oxygen pressure of arterial blood (PaO2) is reduced in severe asthma as a result of ventilation/perfusion mismatch, but it must be interpreted in relation to the concentration of inhaled oxygen to in order to determine the arterial-alveolar (a-A) oxygen gradient.

Option B: Wheezing and its loudness is not a reliable indicator. In severe disease, the wheezing may decrease significantly, resulting in ‘silent-chest’ that is an ominous sign. The wheezing is minimal both with mild attack and just prior to respiratory arrest in very severe attacks.

Option C: In an asthma attack, the severity of the dyspnea is usually a marker of the disease severity, but not the best one because it may be affected by physiologic conditions such as anxiety.

Option D: Forced expiratory volume in one second (FEV1), or peak expiratory flow rate (PEFR) is the best guide to assess the severity, as well as the response to treatment. Of these two, FEV1 is the best single indicator.

Option E: Pulse rate is usually increased in an asthma attack but it might be a consequence of anxiety or the effect of beta 2 agonist or atropine –like drugs such as ipratropium bromide.

  • http://www.ncbi.nlm.nih.gov/pmc/articles/PMC211718
How well did you know this?
1
Not at all
2
3
4
5
Perfectly
8
Q

Which of the following statements is correct regarding asthma in children?

A. Inhaled bronchodilators are very effective under the age of 12 months.
B. Oral bronchodilators are very effective under the age of 12 months.
C. A spacer with face mask should be used for children aged between 1 to 2 years.
D. Most of the children do not grow out of asthma when they become adults.
E. Giving more than 6 inhaled doses of beta-2 agonist can be life-threatening in severe asthma.

A

C. A spacer with face mask should be used for children aged between 1 to 2 years.

For children aged 1-2 years (or even older)a spacer with face mask provides adequate delivery of the inhaled drug and is a the preferred mode to deliver asthma medicine.

(Options A and B) Bronchodilators, inhaled or oral, are not very effective in children younger than 12 months.

(Option D) Most of the children grow out of asthma in adulthood and very few continue to be asthmatic adults.

(Option E) Giving 6 to 12 inhaled doses of beta-2 agonist isvery safe and appropriate if used in
severe asthma.

  • Therapeutic Guidelines – Respiratory; available from http://tg.au.org
How well did you know this?
1
Not at all
2
3
4
5
Perfectly
8
Q

A 12-year-old boy is brought tothe hospital by his mother with acute shortness of breath and wheezing. He has background history of asthma and uses inhalers on an as-required basis. Which one of the following is the best bedside measure to assess the severity of asthma?

A. Severity of the wheeze.
B. Response to inhalers.
C. The degree of dyspnea.
D. Peak expiratory flow rate.
E. Formal pulmonary function tests

A

D. Peak expiratory flow rate.

The peak expiratory flow rate (PEF, also known as a peak flow) is the maximum rate that a person can exhale during a short maximal expiratory effort after a full inspiration.

In patients with asthma, the PEF correlates reasonably well with the the forced expiratory volume in one second (FEV1). FEV1 (not an option) is the most useful tool for bedside assessment of asthma response to treatment. In the absence of FEV1, PEF is the best option for this objective.

Peak flow meters have a limited role in establishing the diagnosis of asthma. However, they are very useful in measuring the severity of asthma exacerbation, or response to treatment.Peak flow meter readings become higher while patients are recovering and lower when the airways are constricted.

(Option A) Severity of wheeze is not reliable sign of severity of asthma. Severe asthma usually presents with silent chest and low grade wheeze.

(Option B) Response to inhalers and degree of dyspnea are also very unreliable signs to assess theseverity of asthma.

(Option C) Dyspnea is not a reliable predictor of asthma severity because apatient with mild asthma and one with severe asthma may be equally breathless.

(Option E) Formal pulmonary function testing is performed at specialized centers and is used to diagnose asthma. In contrast, peak flow measurement is quick and useful in emergency situation to assess the severity of an asthma attack.

References
* http://www.ncbi.nlm.nih.gov/pmc/articles/PMC211718

How well did you know this?
1
Not at all
2
3
4
5
Perfectly
9
Q

A 6-month-old child, with confirmed diagnosis of asthma, is given 6 puffs of salbutamol during an acute asthma attack without adequate response.Which one of the following would be the next best step in management?
A. Repeat another 6 puffs of salbutamol.
B. Give 4 puffs of ipratropium bromide.
C. Give oral steroids.
D. Give intravenous steroids.
E. Give inhaled steroids.

A

A. Repeat another 6 puffs of salbutamol.

Asthma is less common in children younger than 12 months of age. However, if contracted at this age, it is poorly responsive to bronchodilators.

Oxygen and short-acting beta-2 agonists such as salbutamol are used routinely despite poor response to β2 agonists, because a mild asthma attack can still be managed by salbutamol.

Initially up to 6 puffs for children ages 0-5 years and up to 12 puffs for those 6 years or older are given. 20 minutes is waited and the respond assessed. If the attack does not subside, upto 6 puffs can be given every 20 minutes for one hour. If these measures failed ipratropium bromide would be added (4 puffs by mask and spacer or 250 mcg by nebuliser for children 0-5 years, or8 puffs by mask and spacer or 500 mcg by nebuliser for children older than 6 years.

The role of ipratropium in management of an acute asthma attack is controversial. It should be considered if the asthma is severe and unresponsive to maximum dose of short-acting beta agonists.

Oral prednisolone 1 mg/kg should also be given for the first 3 days after the asthma attack.

Corticosteroids have minimal role in acute management of asthma, but should be considered to prevent relapses. Oral forms of steroids (suspension) are available in Australia.

Once the acute attack subsides, the child should be reviewed daily for revision of management plan.

How well did you know this?
1
Not at all
2
3
4
5
Perfectly
10
Q

A 37-year-old woman presents to the emergency department with shortness of breath. She is a known case of asthma and smokes heavily. On pulseoxymetry, she is found to have an O2 saturation of 85% on room air. Which one of the following would be the next best step in management of this patient?

A. 100% oxygen by Hudson mask.
B. Intubation and mechanical ventilation.
C. 100% oxygen by nasal prongs at 3 litres/minute.
D. Oxygen 28% by Venturi mask.
E. Continuous positive airway pressure.

A

D. Oxygen 28% by Venturi mask.

With an O2 saturation of 85% on room hair, this woman is definitely hypoxemic and requires oxygen supplementation. There is no comment about the patient CO2 status to differentiate whether this patient has type I or type II respiratory failure, but patients with the following conditions, if hypoxemic, are at increased risk of hypercapnia if they are given oxygen with FiO2 between 35-60% (either by nasal cannula or face mask) as the respiratory drive relies on hypoxemia and not hypercarbia:

  1. Known chronic obstructive pulmonary disease or bronchiectasis
  2. Severe kyphoscoliosis or ankylosing spondylitis
  3. A known history of heavy smoking
  4. Severe obstructive sleep apnea
  5. Morbid obesity (BMI more than 40 kg/m2)
  6. Musculoskeletal disorders with respiratory muscle weakness, especially if on home ventilation
  7. Overdose of opioids, benzodiazepines or other respiratory depressant drugs.

Under above-mentioned condition, the first-line oxygen therapy would be oxygen 24-28% through Venturi mask. If Venturi mask is not available, oxygen through nasal prongs at a rate of 1- 2 litres/minute could be an acceptable option.

This woman is asthmatic (obstructive) and smokes heavily. High-flow oxygen is very likely to pose her at the risk of hypercapnia; hence she should be started on low-flow oxygen either through Venturi mask or 1-2 liter/min oxygen through nasal prongs.

How well did you know this?
1
Not at all
2
3
4
5
Perfectly
11
Q

Which one of the following lung cancers is most common in non-smokers?

A. Squamous cell carcinoma.
B. Small cell carcinoma.
C. Adenocarcinoma.
D. Mesothelioma.
E. Large cell carcinoma.

A

C. Adenocarcinoma.

95% of all of bronchogenic carcinomas are directly related to cigarette smoking. If non-smokers develop lung cancer, it is more likely to be of adenocarcinoma type.

How well did you know this?
1
Not at all
2
3
4
5
Perfectly
12
Q

A 57-year-old man presents with progressive shortness of breath and cough for the past 4 months. He has smoked 40 cigarettes a day for the past 30 years and worked in a coal mine for the past 20 years. On chest auscultation, expiratory wheezes are heard over the left lung. The only abnormal finding on chest X-ray is a 3x4 opacity in the lower lobe of the left lung. Which one of the following is the most likely diagnosis?

A. Bronchogenic carcinoma.
B. Coal workers’ pneumoconiosis.
C. Mesothelioma.
D. Chronic obstructive lung disease (COPD).
E. Extrinsic allergic alveolitis.

A

A. Bronchogenic carcinoma.

In this patient, with wheezing over the left lung, indicating local obstruction of the small airways and a single opacity in the left lower lobe, as well as long-standing history of heavy smoking, bronchogenic carcinoma would be the most likely diagnosis.

(Option B) Coal workers’ pneumoconiosis is an occupational interstitial lung disease seen in coal miners. It can present with progressive shortness of breath and respiratory crackles. Chest X-ray shows small round densities in the parenchyma, usually involving the upper half of the lungs.

(Option C) Mesothelioma is a rare malignancy, which is associated with asbestosis and presents with pleural thickening on chest X-ray.

(Option D) With the chest X-ray findings, COPD is it less likely. Chest X-ray findings in COPD are hyper-inflated lungs, flattened hemi-diaphragms if emphysema is dominant and increased pulmonary markings if the chronic bronchitis is a feature.

(Option E) Although there are no typical findings on chest X-ray characteristic of extrinsic allergic pneumonitis, just a single opacity on X-ray makes this diagnosis less likely.

How well did you know this?
1
Not at all
2
3
4
5
Perfectly
13
Q

A 70-year-old man presents with recurrent right-sided pleural effusions and 10kg weight loss in the past 6 months. He has the history of smoking 40cigarettes a day for the past 40 years. Chest X-ray shows pleural thickening with no consolidation in the lung fields. Pleural fluid cytology does not show any malignant cells. Which one of the following is the next best step in management?

A. Thoracoscopy.
B. Repeat the chest X-ray.
C. Permanent intercostal catheter.
D. Diuretics.
E. Fluid restriction.

A

A. Thoracoscopy.

The pleural thickening on chest X-ray, the pleural effusion and significant weight loss, in the absence of any abnormalities in lung fields, are suggestive mesothelioma as the most likely diagnosis.

More than 90% of patients with pleural mesothelioma present with pleural effusion. Cytologic examination of the pleural fluid is diagnostic in only 32% of patient, and suggestive in 56%. If a diagnosis cannot be made based on cytologic exam of the pleural fluid, thoracoscopy-guided pleural biopsy should be performed as the next best step in management. The results are 98% diagnostic.

How well did you know this?
1
Not at all
2
3
4
5
Perfectly
14
Q

A 55-year-old man presents with complains of dry cough and hemoptysis. He had worked in a gold mine for 15 years and in a petrol station for another 9 years. Physical examination is inconclusive. A chest X-ray is obtained that is remarkable for a 2 x 3 cm opacity in the right lung field. Which one of the following would be the next best step in management?

A. Bronchoscopy.
B. Computed tomogram (CT) of the chest.
C. Fine needle aspiration.
D. Ultrasonography.
E. Chemotherapy.

A

B. Computed tomogram (CT) of the chest.

Considering the age and the history of working in gold mine and petrol station, this man has bronchogenic cancer until proven otherwise. For every patient with suspected pulmonary pathology, chest X-ray is always the best initial diagnostic tool. With an opacity in the right lung field, two more steps would be ahead: (1) spiral CT scan of the chest to further visualize the lesion and its extend, and (probably) (2) sputum cytology. Once the lesion is confirmed on the spiral CT, referral specialist for bronchoscopy or open lung biopsy is the most appropriate step.

How well did you know this?
1
Not at all
2
3
4
5
Perfectly
15
Q

Which one of the following is unlikely to cause central cyanosis?

A. Polycythemia.
B. Right-to-left cardiac shunt.
C. Left-to-right cardiac shunt.
D. Acute pulmonary edema.
E. Respiratory depression.

A

C. Left-to-right cardiac shunt.

All the given options are associated with central cyanosis except left-to-right shunts. With left-to- right shunts oxygenated blood goes from the left heart to the right heart and no unsaturated blood enters the systemic circulation to cause cyanosis.

Examples of left-to-right shunt include patent foramen ovale, atrial septal defects, ventricle septal defects and patent ductus arteriosus.

Right-to-left shunting is seen in transposition of the great vessels, persistent truncus arteriosus and tetralogy of Fallot.

Polycythemia, decreased respiration and acute pulmonary edema can cause central cyanosis.

How well did you know this?
1
Not at all
2
3
4
5
Perfectly
16
Q

A 27-year-old asthmatic man presents to the Emergency Department with sudden onset of right- sided pleuretic chest pain. On physical examination, he has a blood pressure of 120/75 mmHg, pulse rate of 94 bpm and respiratory rate of 18 breaths per minute. Chest auscultation reveals diminished breath and hyper-resonance in the right lung. No respiratory distress is noted. A chest X-ray is obtained that is significant for a 20% right-sided pneumothorax. Which one of the following will be the most appropriate next step in management?

A. Admit the patient for observation.
B. Perform thoracotomy.
C. Needle aspiration.
D. Water-sealed catheter thoracostomy.
E. Discharge the patient and review in 24 hours.

A

D. Water-sealed catheter thoracostomy.

Pneumothorax is defined as presence of air in the pleural space. Based on the etiology, pneumothorax can be defined as traumatic or spontaneous.

Traumatic pneumothorax is caused by blunt or penetrating chest traumas or invasive procedures (iatrogenic) such as central vein catheterization, pleural biopsy, etc.

Spontaneous pneumothorax is the term used when the condition occurs in the absence of an apparent trauma or procedure. Spontaneous pneumothorax is classified as primary or secondary based on absence or presence of an underlying lung pathology or chest X-ray abnormalities.

Primary spontaneous pneumothorax (PSP) - spontaneous pneumothorax is primary if all of following criteria are met:
* There is no respiratory finding on exam except those related to the pneumothorax
* There is no history of lung disease
* There is no chest X-ray finding other than those related to the pneumothorax
* The patient is young (≤50 years)
* There is no significant history of smoking

Secondary spontaneous pneumothorax (SSP) - spontaneous pneumothorax is considered secondary if either of the following is present:
* Respiratory findings other than those related to pneumothorax
* History of lung disease such as COPD (the most common cause of SSP), asthma, cystic fibrosis sarcoidosis, TB, lung cancer, chronic bronchitis, bronchiectasis, extra- or intrathoracic restrictive lung disease, etc
* Any chest X-ray abnormality other than those related to pneumothorax
* Age >50 years
* Significant history of smoking

NOTE - the typical patient with PSP is a thin tall man of 20- to 40-years old. PSP is rarely seen after 40 year and almost never after the age of 50; therefore, spontaneous pneumothorax in those older than 50 is considered secondary and treated accordingly until proven otherwise.

Size of pneumothorax - Accurate estimation of the size of a pneumothorax is difficult. There different methods for estimation:

Average interpleural distance (AID) method - approximates the size of a pneumothorax from a PA CXR in standing position by taking the sum of the distances (measured in millimeters) between the ribs and the visceral pleura at the apical level (A), midthoracic (B), and basal level (C), then dividing the sum by 3.

Light index - An upright PA chest X-ray is obtained. The width of the lung and the hemithorax are measured (in centimeters). The percentage of pneumothorax is calculated from the following formula:

Pneumothorax percentage = (1 - (width of the lung)^3 / (width of the hemithorax)^3) x 100

*For example if the width of the lung and the hemithorax are 5cm and 10cm respectively, the percentage of pneumothorax will be:

1 - (5)^3 / (10)^3 = 1 - 0.125 = 0.875 or 87.5% (87.5% of the affected hemithorax is occupied with the pneumothorax)

These methods are difficult to apply in practice and often underestimate the size of the pneumothorax. As a result, some clinicians tend to describe a pneumothorax as large or small, rather than using the percentage.

Chest wall - pleural line distance at the hilum level - British Thoracic Society guidelines define a pneumothorax as small if the distance from chest wall to the visceral pleural line (at the
level of the hilum) is less than 2 cm or large if the distance from the chest wall to the visceral pleural line is 2 cm or greater. Some clinicians prefer 3 cm laterally and 4 cm apically as the threshold to distinguish small and large pneumothoraces.

Symptoms:
* Small pneumothoraces are often asymptomatic.
* Larger pneumothoraces can present with:
-Pleuretic chest pain (pain may be referred to shoulder tip)
-Shortness of breath

Clinical findings associated with pneumothoraces include:
* Decreased breath sounds over the affected area due to decreased air entry
* Hyperresonance over the affected area
* Decreased tactile fremitus
* Tracheal deviation to the affected side

Management:
Primary spontaneous hemothorax (PSP) – management of PSP depends on the presence of the symptoms and/or the size of the pneumothorax:

Consider discharging the patient and review in 24 hours, and every 1-2 weeks until the pneumothorax is resolved if:
* The patient is asymptomatic (or minimal symptoms) AND
* The size of the pneumothorax is less than 15% of the affected lung, or the distance between the chest wall and the visceral plural line is ≤2cm

Consider needle aspiration and REVIEW if:
* The patient is symptomatic (pleuretic chest pain or dyspnoea), OR
* The size of the pneumothorax is ≥15%of the affected lung, or the distance between the chest wall and the visceral plural line >2cm

After needle aspiration, admission and catheter drainage (chest tube) would be indicated as the next best step in management if:
* The aspirated air is ≥3 litres
* The distance between the chest wall and visceral pleural line is still >2cm on a chest x- ray taken 4 hours after needle aspiration
* There is significant shortness of breath

NOTE - the rationale behind less invasive initial management of PSP is based upon the fact that the patients with PSP are young and otherwise healthy, and there is no underlying pathology to perpetuate the pneumothorax or prevent it from spontaneous healing.

Secondary spontaneous pneumothorax (SSP) – in SSP, the underlying lung problem prevents the pneumothorax from spontaneous healing. All patients with SPS need to be admitted to the hospital and undergo:

  • Needle aspiration and REVIEW if there are no significant symptoms AND the size of the pneumothorax is less than 15% of the affected lung (or the distance between the chest wall and the visceral plural line is ≤2cm)
  • Catheter drainage (chest tube) if the patient is symptomatic (pleuretic chest pain or dyspnoea) OR the size of the pneumothorax is ≥15%of the affected lung (or the distance between the chest wall and the visceral plural line >2cm)

Traumatic pneumothorax

Unless the patient is asymptomatic and the size of pneumothorax is <15% (or the distance between the chest wall and the visceral plural line is ≤2cm), he patient should undergo catheter drainage (chest tubes). Asymptomatic patients whose pneumothorax is 15% (or the distance between the chest wall and the visceral plural line is ≤2cm) cab be closely observed for spontaneous resolution of the pneumothorax.

NOTE - pneumothorax of any kind and size should be treated with chest catheter (chest tube) insertions if:
* The patient is undergoing general anaesthesia for any reason
* The patient is planned to be intubated and mechanically ventilated
* The patient is planned to be transported by air (air transport)

With the history of asthma, this patient has SSP until proven otherwise. Furthermore, not only is he is symptomatic (pleuretic chest pain), but the size of the pneumothorax is also large. The best management option for him will be water-sealed chest catheter thoracostomy (chest tube insertion).

How well did you know this?
1
Not at all
2
3
4
5
Perfectly
17
Q

A 68-year-old woman, with history of stage IV chronic kidney disease (CKD) presents to the Emergency Department with sudden-onset acute chest pain and shortness of breath. The chest pain is pleuretic in nature. On examination, she has a blood pressure of 130/90 mmHg, pulse rate of 120 bpm and respiratory rate of 24 cycles per minute. ECG shows no abnormalities. A chest X- ray is obtained that is completely clear. Ventilation/perfusion scan is performed showing decreased perfusion in the lower lobe of the left lung. Which one of the following is the most appropriate next step in management?

A. Low-molecular weight heparin (LMWH).
B. Thrombolysis.
C. CTPA.
D. Unfractionated heparin.
E. Warfarin.

A

D. Unfractionated heparin.

Clinical findings suggest pulmonary embolism (PE) as the most likely diagnosis. Decreased perfusion confirms the diagnosis. V/Q scan can be reliable if there is no pre-existing underlying lung pathology leading to misinterpretation of the result. The validity of the V/Q scan in this scenario can be trusted as there is no abnormal finding on chest X-ray.

Although both unfractionated heparin and low-molecular weight heparin (LMWH) have about the same efficacy for treatment of PE, unfractionated heparin is the preferred option in patients with renal disease because LMWH is excreted renally.

(Option A) LMWH is better avoided in patients with renal disease. If LMWH is considered for treatment, the dose should be adjusted according to glomerular filtration rate (GFR).

(Option B) Thrombolysis is reserved for selected patients with hemodynamic instability, in whom there is no contraindication to thrombolysis.

(Option C) Since V/Q scan has already established the diagnosis, CTPA would not be required. Furthermore, CTPA needs intravenous injection of contrast material that is contraindicated in patients with renal impairment. This is the reason why V/Q scan has been used for the diagnosis of PE in the first place.

(Option E) Warfarin is a part of management after INR reaches the therapeutic dose of 2-3. Warfarin has delayed action, and has initial pro-thrombotic effects. Anticoagulation never starts with warfarin alone.

How well did you know this?
1
Not at all
2
3
4
5
Perfectly
18
Q

A 47-year-old man has developed cough and dyspnea over the past few months. There is no history of smoking or occupational exposure. Marked findings on examination are clubbing of the fingers and inspiratory crepitations. Which one of the following is the most likely diagnosis?

A. Mesothelioma.
B. Sarcoidosis.
C. Pulmonary fibrosis.
D. Lung cancer.
E. Mycoplasma infection.

A

C. Pulmonary fibrosis.

The history and physical findings are characteristic of interstitial lung disease (IDL).

Interstitial lung diseases are a group of heterogenous lung diseases with different clinical, physiologic, radiographic, and pathologic presentations. However, a number of common features justify their inclusion in a single disease category.

IDL is characterized by chronic inflammation and fibrosis of the interstitium and lung parenchyma.

The interstitium of the lung (supporting structure) is the area in and around the small blood vessels and alveoli where the exchange of oxygen and carbon dioxide takes place. Inflammation and scarring of the interstitium, and eventually extension into the alveoli, will disrupt normal gas exchange. Although the progression of IDL may be variable from one disease to another, they have common clinical, radiographic, and spirometric findings:

  • Dyspnea - all patients with IDL develop exertional dyspnea that is the most common complaint bringing them to medical attention
  • Nonproductive cough
  • Bibasal inspiratory crackles
  • Evidence of pulmonary hypertension (in advanced disease) such as increased pulmonic sound, right heart failure, etc.
  • Clubbing (not always) especially in idiopathic pulmonary fibrosis and asbestosis
  • Reticular or reticulonodular pattern (‘ground-glass appearance) on chest X-ray
  • Intrapulmonary restrictive pattern and decreased DLCO on pulmonary function testing

Of the given options, only pulmonary fibrosis as an ILD fits the scenario.

(Option A) Mesothelioma is a rare malignancy of the pleural and peritoneum, highly associated with asbestos exposure. It presents with weight loss, recurrent pleural effusions, and pleural thickening and calcification on imaging. Neither the history, nor the clinical findings is consistent with mesothelioma.

(Option B) Sarcoidosis is a multisystem disease characterized by non-caseating granuloma formation in different organ systems. Pulmonary system is the most common affected system. Although pulmonary sarcoidosis is classified as ILD, most patients with pulmonary involvement (90%) are asymptomatic. Symptoms, if present, resemble those of pulmonary TB including productive cough, hemoptysis, and constitutional symptoms such as weight loss, fever, night sweats and malaise.

(Option D) Absence of smoking in the history makes lung cancer less likely. Moreover, lung cancer is associated with more systemic manifestation such as weight loss.

(Option E) Mycoplasma infection such as TB or atypical pneumonia caused by mycoplasma pneumoniae presents with a completely different picture. Pulmonary TB presents with signs and symptoms such as cough, weight loss, night sweats and malaise. Mycoplasma pneumoniae presents with a prodromal syndrome followed by dry cough, sore throat, and low-grade fever. None of these infections are associated with clubbing.

How well did you know this?
1
Not at all
2
3
4
5
Perfectly
19
Q

A 28-year-old woman presents to the Emergency Department with sudden onset dyspnea and pleuretic chest pain. She is 18 weeks pregnant and has a previous history of deep venous thrombosis (DVT). Which one of the following would be the most appropriate initial investigation in this patient leading to a definite diagnosis?

A. D-dimer assay.
B. Ventilation/perfusion scan.
C. ECG.
D. CTPA.
E. Doppler ultrasound scan.

A

B. Ventilation/perfusion scan.

Investigation for pulmonary embolus (PE) in pregnancy is a complicated issue. PE is the leading cause of death in pregnancy in the developed world. Pregnant women have all the 3 components of the Virchow’s triad that are (1) venous stasis, (2) vessel damage and, (3) hypercoagulable state.

Difficulty and confusion in assessment of PE in a pregnant woman is due to the following:

  • Normal physiological changes in pregnancy such as dyspnea, tachycardia, and leg swelling are also symptoms that PE can present with.
  • Pre-test probability tests such as Wells score, simplified Wells score (SWS) or revised Geneva score, cannot be applied for a pregnant woman because pregnant women were excluded from the analysis group for criteria validation.
  • The d-dimer will start to rise in the second trimester and remain elevated for 4-6 weeks post- partum; therefore, it is not a reliable test to use as a screening tool.

If PE is suspected, it is important to make a definitive diagnosis, as management of a PE during pregnancy needs to include choice of anticoagulation, mode of delivery, and consideration of prophylaxis in future pregnancies. There are debates as to optimal methods of diagnostic work-up for pregnant women with suspected PE. For PE, we use latest guidelines by The Royal Australian and New Zealand College of Radiologists as the most accurate and reliable reference for the exam.

The sudden-onset chest pain and shortness of breath and previous history of DVT make PE a very likely diagnosis. If PE is suspected in a pregnant woman, a chest radiograph (not an option) should be performed as the most appropriate initial step to exclude other causes of the symptoms such as pneumonia, and to determine if V/Q scan or CTPA should be performed as the diagnostic test for suspected PE. If the chest radiograph is clear, VQ scan using a lower dose technique is recommended because it is associated with a lower breast dose than CTPA and a comparably small dose to the fetus.

(Option A) D-dimer levels become elevated in most patients after the first trimester. Normal values for each trimester have yet to be validated in a large patient population to enable them to be used to screen for the possibility of pulmonary embolism during pregnancy in routine clinical practice. Therefore, D dimer is generally not used to screen pregnant women for suspected PE, particularly during the second and third trimester when it is physiologically elevated.

(Option C) Ischemia is among differential diagnoses of chest pain. ECG may be considered to exclude cardiac causes of the chest pain, and also to look for ECG changes favoring PE as a diagnosis. Given the sudden-onset of chest pain and shortness of breath in an otherwise healthy young woman who has risk factors for PE, ECG is not likely to be of significant benefit. The pleuretic nature of the pain, on the other hand, makes cardiac ischemia almost unlikely; however, it may be considered in the initial work-up along with chest radiograph. It is not a priority in this woman though.

(Option D) CTPA is associated with increased risk of breast cancer in women of reproductive age and should be replaced with V/Q scan in this patient group if possible. The most common indication for CTPA instead of V/Q scan is an abnormal chest radiograph.

(Option E) Lower limb venous ultrasound is negative in more than 90% of pregnant women with PE. Lower limb venous ultrasound is not recommended as a first line investigation unless there are clinical symptoms of DVT (e.g., unilateral leg swelling, calf tenderness). This is due to the very low rate of lower limb deep venous thrombosis in pregnant women with PE. Almost no V/Q scans or CTPAs would be avoided in pregnant women with possible PE by performing ultrasound first, because ultrasound is almost invariably negative in the absence of specific leg symptoms in favor of DVT. Therefore, the reduction in radiation exposure to the population of women with possible PE and their fetuses that would result from an “ultrasound first” strategy is negligible. However, with leg symptoms, Doppler ultrasound is the most appropriate initial investigation.

How well did you know this?
1
Not at all
2
3
4
5
Perfectly
20
Q

Which one of the following is the most appropriate initial investigation in a patient with suspected pulmonary embolism?

A. Chest X-ray.
B. CTPA.
C. V/Q scan.
D. D-dimer assay.
E. Doppler ultrasound scan.

A

A. Chest X-ray.

When pulmonary embolism is suspected based on history and clinical findings, chest X-ray is the most appropriate investigation to consider first. There are two objectives; the first one is to exclude other lung pathologies that might have similar presentation. The second is to determine whether a V/Q scan or is CTPA is the most appropriate next investigation, because patients with an abnormal chest X-rays are more likely to have a non-diagnostic V/Q scan than those with a normal chest X- rays.

An ECG would be indicated if there is chest pain which cannot be solely attributed to PE as the cause.

After other possible conditions are excluded by chest X-ray and ECG, a clinical pretest probability (PTP) assessment using Well score system, simplified Wells score, or revised Geneva scoring system should be performed. D-dimer assay is then used, as a screening test, to exclude PE
in patients with low to moderate likelihood of having PE.

(Options B and C) CTPA is considered the imaging modality of choice for definite diagosis of PE, provided that it is available and there is no condition on the patient’s side to make its use undesirable (e.g. pregnancy, women of reproductive age, allergic reaction to contrast media, renal impairment, etc), in which case V/Q should be considered the first-line diagnostic modality.

(Option D) D-dimer assay is indicated in patients with low and moderate pretest probability. A negative D-dimer excludes PE, but positive results will require further assessment with CTPA or V/Q scan. Even if D-dimer is considered, an initial chest X-ray is required to estimate the pretest probability.

(Option E)
Doppler ultrasound is used if there are leg symptoms suggesting a DVT as the source of PE. However, CXR remains the most important initial investigation.

How well did you know this?
1
Not at all
2
3
4
5
Perfectly
21
Q

A 45-year-old man presents to your office concerned about asbestosis. He is a construction worker and gets involved in demolitions of old houses for renovation. He has read smewhere that old houses are a source of asbestos and that asbestosis is a serious condition associated with cancer. He says that he and his coworkers do not use protective measures during the work. Which one of the following would be the most appropriate action to take in this situation?

A. Refer him to respiratory specialist.
B. Chest X-ray now and in 5 years’ time.
C. Inform the authorities about the breach of workplace safety. D. Do nothing.
E. Inform him that asbestos carries a very low risk of cancer.

A

B. Chest X-ray now and in 5 years’ time.

Asbestos has been linked to an increased risk of bronchogenic carcinoma, mesothelioma, gastrointestinal cancers, and asbestosis among occupationally-exposed workers. Adequate screening for asbestos-related diseases does not exist at present. However, some tests, particularly chest X-rays and pulmonary function tests, may indicate that an individual has been overexposed to asbestos. It is important for physicians to become familiar with the operating conditions that are associated with potential asbestos exposure. This is particularly important in evaluating medical and work histories and in performing physical examinations.

The current recommendations include taking a complete history, performing physical exam with close attention to respiratory system and the following diagnostic measures:

  • Chest X-ray – A chest X-ray should be taken for all patients with exposure to asbestos; however, initially, the chest X-rays are often inconclusive because it takes approximately 3- 5 years for radiographic signs of asbestos exposure to be evident on X-rays. It is recommended that chest X-rays be repeated at 3-5 years intervals.
  • Pulmonary function tests (PFTs) – simple PFTs available at general practice level should be applied to recognize the possible restrictive patterns caused by asbestosis.

If the condition is work-related, involvement of a respiratory or occupational physician is needed to consider factors such as the effect of the exposure on other workers, how to control the exposure at the workplace and appropriate use of personal protective equipment.

When a current worker has been identified to have been overexposed to asbestos, measures taken by the employer to eliminate or mitigate further exposure should also lower the risk of serious long- term consequences. The employer is required to institute a medical surveillance program for all employees who are or will be exposed to asbestos at or above the permissible exposure limit (0.1 fiber per cubic centimeter of air).

NOTE - Although most patients with asbestos-related lung disease have a strong exposure history, significant disease can occur with even minimal exposure; therefore, even minor or accidental exposures mandates evaluation.

Referring the patient to respiratory specialist (option A) is not necessary at this stage because he is asymptomatic and does not seem to have any issues right now. The surveillance program can be carried out by the general practitioner.

Informing the case to authorities for the breach of workplace safety (option C) may be considered if there is a significant risk that other employees are at risk of harm inflicted on their health. At this stage, however, the most important step is to perform an initial evaluation and setting a surveillance plan for thes patient.

(Option D) While this patient is in need of assessment and being monitored, doing nothing is the most inappropriate action to take.

(Option E) Asbestos exposure and asbestosis are significant risk factor for bronchogenic cancer and mesothelioma, and telling the patient that such correlation is not strong and there is only a slightly increased risk of cancer is incorrect.

How well did you know this?
1
Not at all
2
3
4
5
Perfectly
22
Q

A 26-year-old woman presents to the Emergency Department with dyspnea and sudden-onset chest pain that worsens with respiration. On examination, she has a blood pressure of 130/95 mmHg, pulse rate of 110 bpm and respiratory rate of 24 breaths per minute. Her family history is significant for protein C deficiency in her mother and older sister. Which one of the following is the most diagnostic investigation in this patient?

A. Doppler ultrasound scan.
B. Chest X-ray.
C. V/Q scan.
D. CTPA.
E. D-dimer assay.

A

C. V/Q scan.

Family history of thrombophilia is associated with increased risk of venous thromboembolism (moderate risk factor). With this risk factor present in the history, acute-onset pleuretic chest pain (worsened by respiration) and shortness of breath, pulmonary embolism (PE) would be the most likely possibility.

Not all patients presenting with possible symptoms of PE need to undergo imaging tests such as computer tomography pulmonary angiography (CTPA), V/Q scan, Doppler ultrasound, etc. Decision as to whether this tests are performed depends on pretest probability of PE. A reasonable way to stratify patient risk is to use one of the validated clinical decision rules (CDR). Simplified Wells Score (SWS) is one of recently validated ones.

SWS takes into account different variables to generate a score. In SWS, each element in the CDR is scored 1 point. The cut-off value distinguishing between “PE likely” and “PE unlikely” is 2, so patients with a score of 1 or 0 are classified as “PE unlikely” (chance of having PE ≤10%), while those with scores 2 or above are classified as PE likely.

Elements for calculation of SWS and an algorithmic approach based on SWS score are shown in the following table and diagram respectively:

See table below.
See flow chart on Simplified Well’s Score on Page 1820.

The question however asks about the most diagnostic test and not the approach sequence. CTPA and V/Q scan are used to confirm the diagnosis of PE. Diagnostic accuracy of CTPA and V/Q scan is similar. However, CTPA detects clots in smaller vessels. CTPA may have the advantage of widespread availability where V/Q scanning may not be available outside working hours. Radiation dose of V/Q is significantly less than CTPA, making V/Q scan the preferred option for women of reproductive age (< 55 years) to avoid the risk of breast cancer associated with higher radiation exposure in CTPA.

(Option A) Doppler ultrasound, when leg symptoms associated with deep vein thrombosis (DVT) are absent, is not very helpful to establish a diagnosis.

(Option B) Chest X-ray is the initial imaging in all patients with suspected PE to exclude other pathologies and guide further assessment. However, the chest X-ray findings are often non-specific and non-diagnostic.

(Option D)
CTPA is a very useful diagnostic test for PE, but not preferred over V/Q scan in this young woman because it is associated with more radiation exposure and increased risk of breast cancer.

(Option E) D-dimer assay is used to exclude PE in patients, who are unlikely to have PE, based on pretest probability. It is sensitive enough to be used as a screening test, but not specific to establish a diagnosis.

How well did you know this?
1
Not at all
2
3
4
5
Perfectly
23
Q

A 45-year-old man, who had a hip replacement surgery 2 weeks ago, presents with sudden-onset chest pain and shortness of breath. Pulse oxymetry reveals an oxygen saturation of 89% on room air. On physical examination, his blood pressure is 100/75mmHg, heart rate 113 bpm, respiratory rate 23 breaths per minute and temperature 37.3°C. Which one of the following is the most appropriate next step in management of this patient?

A. CTPA.
B. V/Q scan.
C. ECG.
D. Chest X-ray.
E. D-dimer assay.

A

C. ECG.

Both major surgery and possible immobilization of longer than 3 days are strong risk factors for thromboembolism; therefore, with sudden-onset chest pain, shortness of breath and hypoxia, PE tops the list of differentials. CTPA and V/Q scan are used to confirm the diagnoses; however, not every patient with clinical manifestations of PE and a suggestive history should undergo imaging studies, unless other potential causes of the presentation have been excluded.

Although the acute onset of the chest pain is not in favor of ischemic heart problems such as acute coronary syndrome, an ECG should be considered the very first step to exclude cardiac problems as a cause.

Moreover, some ECG findings such as Q1S3T3 (Q wave in lead I, prominent S wave and inverted T-wave on lead III) or T –wave inversion in leads V1 through V4, although non-specific, might support PE as diagnosis (but not diagnostic).

How well did you know this?
1
Not at all
2
3
4
5
Perfectly
24
Q

A 27-year-old woman presents to your clinic with fever and cough since yesterday. On examination, she looks ill and unwell. Her blood pressure is 110/70 mmHg, pulse rate 110 bpm, respiratory rate 22 breaths per minute and temperature 38.7°C. Pulse oxymetry shows oxygen saturation of 89% while on room air. Which one of the following is the next best step in management?
A. Continuous positive airway pressure (CPAP).
B. 60% Oxygen through nasal prongs.
C. 100 % Oxygen through Hudson’s mask.
D. Oxygen through Venturi mask – 28%.
E. Intubation and ventilation.

A

C. 100 % Oxygen through Hudson’s mask.

This patient has clinical features of pneumonia. With an oxygen saturation of 89% (less than 90%), she should be considered hypoxemic until ABG results are available.

Patients with serious acute respiratory illnesses such as acute asthma, pneumonia, lung cancer, pulmonary embolism, pleural effusion, and pneumothorax require moderate levels of supplemental oxygen. If the patient is hypoxemic on pulse oxymetry (oxygen saturation less than 90%). This may be 100% oxygen by nasal cannulae at 2 to 6 L/minute or a simple face mask at 5 to 10 L/min initially, but if the oxygen saturation is less than 85% and the patient is not at risk of hypercapnic respiratory failure, treatment should be commenced with 100% oxygen through reservoir mask at 10 to 15 L/min.Mechanical ventilation such as CPAP and intubation and ventilation are considered if the patient fails to maintain the target O2 saturationdespite adequate supplementation of oxygen with other delivery systems such as nasal cannulae, simple face mask, reservoir masks or no-rebreather masks

Oxygen through Venturi mask is first-line option for patients at risk developing hypercapnic respiratory failure (type II). These patients include the following:
1. Known cases of chronic obstructive pulmonary disease or bronchiectasis
2. Severe kyphoscoliosis or ankylosing spondylitis
3. A known history of heavy smoking
4. Severe obstructive sleep apnoea
5. Morbid obesity (BMI more than 40 kg/m2)
6. Musculoskeletal disorders with respiratory muscle weakness, especially if on home
ventilation
7. Overdose of opioids, benzodiazepines or other respiratory depressant drugs.

NOTE - ‘Hudson mask’ is a confusing name. ‘Hudson RCI’ is the manufacturer and produces a variety of oxygen delivery devices including nasal prongs, simple face masks, reservoir face masks and non-rebreather masks, but historically the Hudson mask is the inappropriate equivalent of simple face masks.

References
* https://www.brit-thoracic.org.uk/document-library/
* http://www.ncbi.nlm.nih.gov/pmc/articles/PMC111390
* http://www.respiratoryupdate.com/members/Levels_of
* Therapeutic Guidelines – Respiratory; available on http://www.tg.org.auOxford Handbook of
Clinical Medicine – 8th Edition – pages 180-181

How well did you know this?
1
Not at all
2
3
4
5
Perfectly
24
Q

A 12-year-old boy is brought to the emergency department with a progressively worsening asthma attack. He has past medical history of previous asthma attacks and recurrent chest infections.
Which one of the following, if present, would indicate life-threatening asthma?
A. Wheeze.
B. Cyanosis.
C. Speaking in sentences.
D. Peak expiratory flow more than 75%.
E. Oxygen saturation 95%.

A

B. Cyanosis.

If present, cyanosis indicates a life-threatening asthma episode. Other indicators of severity are FEV1< 50%, peak expiratory flow < 50%, the patient speaking in words, physical exhaustion, pulse rate> 120bpm, oxygen saturation< 92%, and significant accessory muscles use.

(Option A) Wheeze is not a reliable indicator of the severity of an asthma attack. In a severe attack, the wheeze disappears and results in the ‘silent chest’ just prior to respiratory collpase. A loud wheeze may not be heard in the noisy environmet of the emergency department.

(Option C) A patient can speak in sentences in mild attack, in phrases in moderate and in words in severe attacks.

(Option D) Peak expiratory flow meter is usually more than 75% in mild asthma, 50-75% in moderated and < 50% is severe asthma attacks.

(Option E) Oxygen saturation of more than 94% is seen in mild, between 92-94% in moderate and < 92% in a severe attacks.

How well did you know this?
1
Not at all
2
3
4
5
Perfectly
25
Q

A 5-year-old boy is brought to your practice, by his mother, because of intermittent attacks of wheezing, which have been occurring4-6 times a year over the past 2years. These attacks are usually precipitated by viral upper respiratory tract infections and last 2-3 days. The symptoms respond to treatment with inhaled salbutamol. There are no interval symptoms. His older brother had flexural eczema until the age of 18 months. Which one of the following is the most appropriate management of this child?

A. Inhaled sodium cromoglycate.
B. Inhaled fluticanose.
C. Oral montelukast.
D. Oral prednisolone when symptomatic.
E. Inhaled salbutamol when symptomatic.

A

E. Inhaled salbutamol when symptomatic.

With infrequent episodes of wheezing and absence of interval symptoms, this child is most likely to have mild intermittent asthma. Mild asthma in children and adults is managed by short-acting β2 agonists such as salbutamol.

Inhaled fluticasone or other corticosteroids are not used for mild intermittent asthma, and is reserved for more severe forms of asthma.

Montelukast is often considered and tried for a period of 2-4 weeks before the child is started on inhaled corticosteroids in an attempt to spare the child from corticosteroids and their potential adverse effects.

References
* http://www.asthmahandbook.org.au/uploads/Australia
* Therapeutic Guidelines – Respiratory; available from http://tg.org.au

26
Q

Which one of the following is correct about combination of long-acting β2 agonists with inhaled corticosteroids in children with asthma?
A. It reduces the frequency of severe exacerbations.
B. It may increase the frequency of severe exacerbations.
C. It has no effect on asthma.
D. It is excellent for exercise-induced asthma.
E. Long acting β2 agonists decrease the need for inhaled steroids.

A

B. It may increase the frequency of severe exacerbations.

Long-acting beta2 agonists are currently overprescribed in children. They are also often used inappropriately as first-line therapy and are not recommended for children aged five years or less.

Due to the paucity of pediatric clinical trials, the evidence for the efficacy and safety of long-acting beta2 agonists in children is limited. There is little evidence that they reduce the risk of severe exacerbations and some evidence that they may actually increase the risk.

The regular use of long-acting beta2 agonists may also result in a loss of protection against exercise-induced bronchoconstriction, and the development of tolerance to short-acting beta2 agonists. Long-acting beta2 agonists are only one option for children whose asthma is not adequately controlled with inhaled corticosteroids alone – the other options being an increase of inhaled corticosteroid dose or the addition of a leukotriene receptor antagonist.

For children whose major ongoing symptoms are activity related, the addition of a leukotriene receptor antagonist is the preferred option.

(Option A) There is little evidence for efficacy of LABAs in reducing the acute asthma exacerbation, and in fact, some studies have shown that LABA use can increase the frequency of asthma exacerbations in children.

(Option C) Evidence has shown some effectiveness for LABAs in adult patients and under some circumstances for children, often as measure of last resort. But it is associated with the problems like possible increased frequency of acute exacerbations and tolerance to short-acting beta2 agonists.

(Option D) First-line medications for prevention of exercise-induced asthma are short-acting beta2 agonists (SABA) such as salbutamol, 20 minutes before the anticipated exercise. LABAs not only are inappropriate for this purpose, but they also have been shown to reduce the protective effects of SABAs.

(Option E) LABAs do not decrease the need for preventers such as inhaled corticosteroids (ICS). Low to medium doses of ICS are best preventers to consider for children with asthma as first-line.

Combination therapy with ICS and LABA should only be considered if other measure such as maximum dose of ICS and leukotriene receptor antagonists (e.g. Montelukast) fails or is inadequate.

27
Q

A 10-year-old male child is brought in by his mother with complaint ofongoing exercise-induced asthma, despite being on maximum dose of inhaled corticosteroids. Which one of the following is the most appropriate next step in management?
A. Refer to respiratory physician.
B. Add leukotriene receptor antagonists.
C. Cease inhaled corticosteroids and start intravenous steroids.
D. Add long-acting β2 agonists.
E. Add short-acting β2 agonists.

A

B. Add leukotriene receptor antagonists.

For children aged 4-16 years, who are already taking an inhaled corticosteroid, but their exerciseinduced symptoms do not improve, addition of regular leukotriene receptor antagonist (montelukast, zafirlukast) is the next best step in management.

  • http://www.asthmahandbook.org.au/uploads/Australia
28
Q

A 69-year-old man is brought to the emergency department with severe shortness of breath and cough. He has had 2 similar episodes in the past 6 months. His background history includes smoking of 30 cigarettes a day for the past 25 years. On examination, he is in moderate distress with a blood pressure of 110/75mmHg, pulse of 120 bpm and respiratory rate of 34 breaths per minute. he is started on oxygen 24% through Venturi mask. A bedside pulseoxymetry shows oxygen saturation of 85%. Which one of the following is the next best step in management?

A. Chest X-ray.
B. ABG.
C. Serum electrolytes.
D. Full blood count.
E. Assisted ventilation with CPAP

A

B. ABG.

The clinical findings, the history of smoking, and the oxygen saturation of 85%, are suggestive of an acute exacerbation of COPD. When hypoxemia is suspected on pulse oxymetry, an arterial blood gas (ABG) analysis is mandatory as the next best step to not only confirm the hypoxia, but also to assess the pH and partial arterial pressure of carbon dioxide (PaCO2) for further managment planning, as well as having a baseline for monitoring the response to treatment.

Chest X-ray (option A) , serum electrolytes (option C) and full blood count (option D) are part of the initial assessment, but the most important step remains ABG.

Assisted ventilation with CPAP (option E) or intubation and ventilation are considered if the patient remains hypoxic and hypercapnicdespite judicious administration of oxygen or if the patient is in severe respiratory distress (e.g. accessory muscle use, etc).

  • https://www.mja.com.au/journal/2003/178/6/copdx-pl
  • http://www.uptodate.com/contents/management-of-exa
  • Therapeutic Guidelines – Respiratory; available on: http://tg.org.au
29
Q

A 67-year-old man withbackground history of smoking for the past 40 years and chronic
obstructive pulmonary disease (COPD) presents with acute shortness of breath, pleuretic chest pain, increased mucus production and severe cough. Last week, he has had an episode of gout in hisfirst metatarsophalangeal joint, for which he was started on indomethacin 75 mg, 8-hourly.
Severe pain made him bedbound for 5 days. On examination, he has a blood pressure of 130/95 mmHg, pulse rate of 110 bpm, respiratory rate of 34 breaths per minute and temperature of 37.5°C.
Which one of the following is the next best step in management?
A. Chest X-ray.
B. Arterial blood gas (ABG) analysis.
C. Computed tomography pulmonary angiography (CTPA).
D. Ventilation/perfusion scan.
E. Chest CT scan.

A

B. Arterial blood gas (ABG) analysis.

Pleuretic chest pain, tachycardia, and tachypnea and the history of immobility is highly suggestive of pulmonary embolism (PE). PE on the other hand, can precipitate a COPD exacerbation that justifies the increased amount of sputum production, hypoxia and hypercapnia. Under these circumstances, an ABG is always the next best investigation to check for hypoxia (the most important concern in both PE and COPD), worsened hypercapnia due to COPD exacerbation, and the blood pH.

A chest X-ray (option A) should be obtained for this patient to investigate any other
underlying lung pathology that might, other than the PE, has led to the condition, but immediate pulseoxymetry and ABGalways come first in priority.

Ventilation/perfusion scan (option D)orCTPA (preferred) (option C) are diagnostic tools to
establish the definite diagnosis of PE. They should be performed to prove the presence if PE prior to treatment with anticoagulation.

A conventional CT scan of the chest (option E) has no role in management of this patient for now.

With the underlying COPD and baseline chest X-ray abnormalities, ventilation/perfusion scan is unlikely to be of diagnostic value. Conventional CT scan of the chest would add no relevant pieceof information.

  • Therapeutic Guidelines – Respiratory; available from http://tg.org.au
30
Q

A 60-year-old man is brought to the emergency department by paramedics because of severe
shortness of breath. On his arrival, a bedside pulse oxymetry shows an O2 saturation of 85% on room air. His son states that this is the fifth time he is admitted to the emergency department due to the same problem. On further inquiry, he is found to have smoked 60 packs per year for the past 30 years. He is given oxygen 28% by Venturi mask. 30 minutes later an arterial blood gas (ABG) reports his PaO2 and PaCO2 68 mmHg and 60 mmHg respectively. The O2 saturation is 88% while
on oxygen. Which of the following is the next best step in the management of this patient?
A. Stop oxygen and check ABG after 30 minutes.
B. Continue oxygen until normal PaO2 is achieved.
C. Immediate intubation and ventilation.
D. Provide immediate antibiotic coverage against gram negative bacteria.
E. Start intravenous aminophylline infusion.

A

B. Continue oxygen until normal PaO2 is achieved.

Oxygen therapy, bronchodilators, systemic corticosteroids and antibiotic are mainstay of therapy in an acute exacerbation of COPD. The main goal of oxygen therapy is delivering adequate oxygen, while preventing from hypercapnia and acidosis. Since this patient has not deteriorated since presentation and commencement of oxygen therapy, maintaining the same rate of oxygen delivery is the best appropriate management.

Adequate oxygenation must be assured, even if it leads to acute hypercapnia. Hypercapnia is
generally well tolerated in patients whose arterial carbon dioxide tension (PaCO 2
) is chronically elevated. However, mechanical ventilation may be required if hypercapnia is associated with depressed mental status, profound acidemia, or cardiac dysrhythmias.

Stopping oxygen causes the patient’s condition to worsen again and is not an appropriate option. Immediate intubation and ventilation is not necessary at this point because the patient is rather stable, with no signs of hypercapnia as mentioned above

Antibiotic are indicated if a bacterial infection is suspected base on the presence of (1) increased dyspnea, (2) increased sputum volume, or (3) purulent sputum. The most common culprits in bacterial infections are streptococcus pneumoniae, hemophilus influenza and moraxella catarrhalis. These are gram positive organisms and are adequately covered by amoxicillin or doxycycline.

  • Therapeutic Guidelines – Respiratory; available on: http://tg.org.au
  • http://www.uptodate.com/contents/management-of-exa
31
Q

A 68-year-old nursing home resident man, with long standing chronic obstructive pulmonary
disease (COPD), has been brought by ambulance to the emergency department because of an exacerbation of his condition. En route to the emergency department, he was severely short of breath and was given oxygen 10 L/min via nasal canula. On examination, he is drowsy and disoriented. Which one of the following could be the most likely result of his arterial blood gas
(ABG)?
A. Ph=7.29, PaCO2=65mmHg, PaO2=85mmHg.
B. Ph=7.15, PaCO2=50mmHg, PaO2=68mmHg.
C. Ph=7.25, PaCO2=25mmHg,PaO2=100mmHg.
D. Ph=7.35, PaCO2=40mmHg, PaO2=40mmHg.
E. Ph=7.45, PaCO2=85mmHg, PaO2=40mmHg.

A

A. Ph=7.29, PaCO2=65mmHg, PaO2=85mmHg.

The clinical picture is consistent with carbon dioxide (Co2) narcosis. Agitation, confusion, tremors, convulsions, and possible coma may occur if blood levels of carbon dioxide rise to 70 mmHg or higher. Individuals with chronic obstructive pulmonary disease (COPD) can have CO2 narcosis with no symptoms other than confusion and/or drowsiness, because they have already developed tolerance to elevated amounts of CO2. When ventilation is sufficient to maintain a normal PaO2 in the arteries, the carbon dioxide partial pressure is generally expected to be near 40 mmHg.

This patient has been on high-flow oxygen so his blood oxygen content is expected to be normal or even high. On the other hand, this patient is dependent on hypoxia rather than hypercapnia as the main stimulant of respirtory drive. With excessive oxygenation respiration will be suppressed and he is likely to have CO2 retention and increased blood CO2. CO2 retention also results in repiratroy
acidosis. Of the options, option A is consistent with these findings.

TOPIC REVIEW
If apatient with acute exacerbation of COPD is hypoxemic, oxygen should be used as the most
appropriate next step in management to maintain the arterial oxygen saturation at about 90% (88%-92%). Oxygen therapy startswith 2L/minute through nasal cannula (prongs) or 24- 28% oxygen viaVenturi mask.

Serial ABGs are used to monitor for hypercapnia or acidosis and accurate measurement of blood oxygen content.

With high flow or high concentration of oxygen some patients may develop hypercapnia, probably because supplemental oxygen contributes to the following:
1. Increasing the perfusion/ventilation mismatch
2. Suppression of respiratory drive

Ventilatory support may be needed if hypercapnia develops or worsens despite:
- Optimal drug therapy
- Treatment of other complications
- Attempt to minimize inspired oxygen while maintaining adequate O2sat≥88% to 90%

Non-invasive ventilatory support (NIVS) with CPAP or BiPAP may avoid the need for intubation.

Supplemental oxygen should be maintained at minimum levels required for adequate oxygen
saturation, peripheral oxygen delivery, and stable pressure of PaCO2 and pH. Adjust the oxygen or decide for ventilation according to the following table:

  • Therapeutic Guidelines – Respiratory; available from http://tg.org.au
  • http://www.uptodate.com/contents/management-of-exa
32
Q

A 72-year-old man, who is a known case of chronic obstructive pulmonary disease (COPD),
presents to the emergency department with severe shortness of breath and cough. Based on the diagnosis of a COPD exacerbation, he is started on oxygen 28% by Venturi mask. After one hour, an ABG is obtained that shows a PaO2 of 42mmHg and a PaCO2 of 68mmHg. Which one of the following is the next best step in management?
A. Increase the oxygen flow and use face mask.
B. Assisted ventilation.
C. Decrease the oxygen flow.
D. Continue with the same amount of oxygen and repeat the ABG after one hour.
E. Add inhaled short-acting β2 agonists.

A

B. Assisted ventilation.

Thispatient has been started on standard oxygen therapy with 28% oxygen viaVenturi mask, but has not responded to the treatment, and now has a PaO2 of less than 60 mmHg. The next step for this patient would be assisted ventilation either by CPAP or BiPAP (preferred), or intubation and mechanical ventilation.

Recommendations for oxygen adjustment for patients with an episode of COPD exacerbation is summarized in the following table: - See table below.

(Option A)Patients with COPD are dependent on hypoxia rather than hypercapnia to increase their respiratory drive. Increasing the oxygen flow in this patient may initially result in increased levels of blood oxygen, which results in decreased respiration and deterioration of their hypercapnia.

(Option C) Decreasing the oxygen flow will lead to more hypoxia and deterioration of the condition.

(Option D) Current oxygen flow has failed to provide adequate oxygenation for this patient and has also resulted in further hypercapnia. Continuation of treatment with the same amount of oxygen will not benefit the patient.

(Option E) Adding β2 agonists and anticholinergic agents are an essential part of the management plan, but should be administered with nebulisers. Inhaled drugs are of trivial benefit in COPD exacerbations.

  • Therapeutic Guidelines – Respiratory; available from http://tg.org.au
  • http://www.uptodate.com/contents/management-of-exa
33
Q

A 71-year-old man, known case of COPD for the past 8 years, is brought to the emergency
department by ambulance due to severe difficulty in breathing. On the way to the hospital, he is given oxygen 100% by nasal cannula. Upon arrival to the emergency department, his oxygen saturation is 81% and his shortness of breath is even worse,according to the paramedics. Which
one of the following would be the next best step in management?
A. Intubation.
B. Cessation of oxygen.
C. Increasing the oxygen flow.
D. Decreasing the oxygen flow.
E. Starting the patient on bronchodilators.

A

D. Decreasing the oxygen flow.

COPD patients has type II respiratory failure, which by definition is hypoxia (PaO2<60mmHg (8.0 kPa)) and hypercapnia (PaCO2> 50mmHg (6.6 kPa)). Under normal condition, both decrease in PaO2 and increase PaCO2 stimulate respiratory centersin the brainstem and increase the respiratory drive as a compensatory mechanism, but in chronic type II respiratory failure, such as in COPD, sustained exposure to high levels of PaCO2, desensitizes respiratory centersto elevated concentrations of CO2 in the blood. Administration of high-concentration oxygen to these patients
decreases the respiratory drive and results in more hypercapnia, dyspnea, paradoxically decreased oxygen saturation and respiratory acidosis.

In such patients with deteriorating PaO2 and oxygen saturation despite being on oxygen the next best step in management is to reduce the delivery of oxygen, provided that the patient is not at risk of imminent respiratory arrest or severe acidemia (PH<7.3) in which immediate case assisted ventilation should be considered.

If patient remains hypoxemic despite adequate controlled oxygen delivery (e.g. by Venturi mask)
assisted ventilation such as CPAP or intubation (option A) and mechanical ventilation should be
considered next.

Both increasing the oxygen flow (option C) and cessation of oxygen (option B) will increase the
hypoxemia deteriorate the patient’s condition as the first decreases the respiratory drive even more while the second deprives him of oxygen.

(Option E) While the respiratory drive is suppressed, administration of bronchodilators is not likely to be beneficial for this patient.

References
* Therapeutic Guidelines – Respiratory; available on: http://tg.org.au
* UpToDate - Management of exacerbations of chronic obstructive pulmonary disease

34
Q

A 57-year-old man presents to the emergency department with severe shortness of breath and tachypnea. As a part of emergency evaluation, an arterial blood gas (ABG) analysis is performed, the result of which are as follows:
PH: 7.2 (normal 7.35-7.45)
PaCo2: 48 (normal 35-45 mmHg)
PaO2: 50 (normal 80 -100 mmHg)
Which one of the following is the next best step in management?
A. 100% oxygen by Hudson mask.
B. 28% oxygen by Venturi mask.
C. Intubation and ventilation.
D. Positive pressure ventilation.
E. Oxygen 6 litres/minute by nasal prongs.

A

D. Positive pressure ventilation.

In case ofrespiratory failure, oxygen supplementation is the next best step in management. This can be achieved either by high concentration oxygen through nasal cannulae, simple face mask, reservoir masks or non-rebreathing masks in type I (hypoxemic respiratory failure) or lowconcentration oxygen through Venturi mask in type II (hypercarbic) respiratory failure or for those patient at increased risk of type II respiratory failure.

As a rule, however, patients with the following conditions should be supported with assisted
mechanical ventilation, either with non-invasive measures (CPAP, BiPAP) (preferred option) or
conventional intubation or mechanical ventilation as the last resort if there is:
- persistent or worsening hypoxemia despite supplemental oxygen or inability to achieve an O2
- saturation above 90% OR
- Worsening hypercapnia (PaCO2>50 mmHg) OR
- Worsening respiratory acidosis (PH<7.25) OR
- Clinical manifestations of respiratory distress such as:

  • Severe dyspnea
  • Tachypnea (RR>30)
  • Nasal flaring
  • Accessory muscle use
  • Tracheal tugging
  • Recession of suprasternal and intercostal spaces
  • Pulsus paradoxus
  • Diaphoresis
  • Paradoxic motion of the rib cage and abdomen

When the patient is not conscious enough to maintain their airway patency or when there is
productive cough, invasive mechanical ventilation (intubation) should be considered first.

This patient has a PH of 7.2 and requires positive pressure assited ventilation by C-PAP or BiPAP as the most appropriate next step in management.

  • http://www.merckmanuals.com/professional/critical_
  • Davidson’s Principles and Practice of Medicine – 20th Edition – pages 193-195 and 669
34
Q

Mr. Browny brings her wife to the emergency department because he found her walking in the front yard in her night gown and speaking to herself incoherently. She is 68 years old and has the established diagnosis of chronic obstructive pulmonary disease (COPD) made 3 years ago. She has had 3 episodes of COPD exacerbation in the past 18 months, for which she has been hospitalized. On examination, she looks confused, with a mini mental status exam (MMSE) score of 24. Which one of the following investigations is more likely to establish the cause of her presentation?
A. ABG.
B. Chest X-ray.
C. CT scan of the head.
D. Urinalysis.
E. Basic metabolic panel.

A

A. ABG.

Patients with exacerbation of COPD have deterioration of the pre-existing hypoxia and hypercapnia, both of which can cause a confusional state and delirium. With COPD in history, the initial investigation would always be an arterial blood gas (ABG) analysis to for hypoxia, reflected by a decreased PaO2 level, or hypercapnia indicated by an elevated PaCO2.

Management of hypoxia always takes precedent over correction of hypercapnia.

(Option B) A chest X-ray is always required for patient with exacerbations of COPD to look for an underlying cause for the deterioration, but this is not the first step.

(Option C)CT scan of the head may be indicated to exclude intracranial bleeding, tumors, etc, but as mentioned earlier, ABG always comes first.

(Option D) Urinalysis is considered when urosepsis is suspected. Although there is no comment about ‘fever’, it should be borne in mind and investigated, as a part of workup planin elderly patients with new-onset confusion.

(Option E) Electrolyte disturbances (e.g. hypo/hypernatremia) are other possible causes of suppressed mental status and confusion, and should always be investigated promptly.

  • Therapeutic Guidelines – Respiratory; available on: http://tg.org.au
  • UpToDate - Management of exacerbations of chronic obstructive pulmonary disease
35
Q

A 78-year-old man is found unconscious in the backyard garden by his wife and is brought to the emergency department. He was diagnosed with chronic obstructive pulmonary disease (COPD) 7 years ago and is currently on inhaled ipratropium bromide. He also has ischemic heart disease, for which he takes aspirin, verapamil and statins. On examination, he is drowsy and has a blood pressure of 110/70 mmHg, pulse rate of 100 bpm, and respiratory rate 24 breaths per minute. On chest examination, bilateral wheezing is noted. Bedside pulseoxymetry shows oxygen saturation of 85% on room air. An ABG showsPaO2 of 60 mmHg and PaCO2 of 50 mmHg. Which one of the following is the most likely cause of this clinical picture?
A. Infection.
B. Electrolyte disturbances.
C. Sedative overdose.
D. Left ventricular failure.
E. Right ventricular failure.

A

A. Infection.

The most likely explanation to this clinical presentation is either hypoxia (the most important concern) or hypercapnia and CO2 narcosis, or both in the setting of a COPD exacerbation.All the given options can be potential causes of the decompensation of the COPD in this man, but viral infections remain the most common underlying etiology

Electrolyte disturbances (option A) are another important cause of confusion and drowsiness, but not as common as viral infections.

(Option C)Although sedatives overdose, by suppressing the respiratory drive, can lead to both hypoxia and hypercapnia, there is nothing in the history pointing towards thisetiology as a
possibility.

(Option D)If the left ventricular failure was the cause more pronounced presentation such as
pulmonary edema (basal crackles), and S3 gallopwere expected.

(Option E) With right ventricular failure, signs of increased venous pressure such as a distended
jugular vein, peripheraledema, and hepatojugular refluxwould be present.

  • Therapeutic Guidelines – Respiratory; available on: http://tg.org.au
  • UpToDate - Management of exacerbations of chronic obstructive pulmonary disease
36
Q

Which one of the following is the most common cause of secondary pulmonary hypertension and corpulmonale?
A. Emphysema.
B. Bronchiectasis.
C. Pulmonary embolism.
D. Pneumothorax.
E. Foreign body

A

A. Emphysema.

Emphysema as a component of chronic obstructive pulmonary disease (COPD) is the most common cause of cor pulmonale worldwide and in Australia.

Causes of cor pulmonale are as follows:
Acute
- Pulmonary embolism
- Exacerbation of chronic cor-pulmonale
- Acute respiratory distress syndrome(ARDS)

Chronic
- Chronic obstructive pulmonary disease
- Primary pulmonary hypertension
- Uncontrolled Persistent Asthma
- Loss of lung tissue following trauma or surgery
- End stage pneumoconiosis
- Sarcoidosis
- T1-4 vertebral subluxation
- Untreated obstructive sleep apnea
- Interstitial lung disease
- Sickle cell anemia
- Bronchopulmonary dysplasia (in infants)
- Severe thoracic kyphoscoliosis

References
* http://www.uptodate.com/contents/cor-pulmonale
* http://www.merckmanuals.com/professional/cardiovas

37
Q

Which one of the following is the best indicator of development of right heart failure in a patient with chronic obstructive pulmonary disease (COPD)?
A. Hypoxia.
B. Hypercapnia.
C. Reduced forced vital capacity.
D. Increased jugular venous pressure.
E. Hepatomegaly.

A

D. Increased jugular venous pressure.

Right heart failure is a common complication of long-standing COPD. The underlying
pathophysiology is constriction of pulmonary vasculature resulting in pulmonary hypertension.

Pulmonary hypertension leads to right ventricular failure with the following features:
- Increased jugular venous pressure
- Hepatomegaly
- Ascites (rarely)
- S3 gallop (right-sided)
- Edema, particularly that of the lower limb

Of the given options, raised JVP is the most specific indicator of right heart failure.

(Options A and B) Although hypoxia and hypercapnia may become worse with right heart failure, they often pre-existas a result of COPD.

(Option C) Reduced FVC can be seen in severe obstructive and restrictive lung diseases. It is not a sign of right heart failure.

(Option E) Hepatomegaly is often a latesign of right heart failure, but it may also be present in
other conditions such as portal hypertension, and is not specific to right heart failure.

References
* Medscape - Cor Pulmonale
* MSD Manuals - Cor Pulmonale

38
Q

A 72-year-old woman presents to your clinic for medical evaluation. Her past medical history
includes chronic obstruction pulmonary disease (COPD), hypertension and smoking of 30 cigarettes a day for the past 40 years. On examination, there is bilateral ankle edema, raised jugular vein pressure and positive hepatojugular reflux. Chest exam reveals loud P2 and, globally decreased air entry and fine bilateral crackles. Which one of the following is the most likely diagnosis?
A. Right heart failure.
B. Exacerbation of chronic obstructive pulmonary disease.
C. Pneumonia.
D. Cor pulmonale.
E. Left heart failure.

A

D. Cor pulmonale.

With clinical findings of right heart failure (peripheral edema, raised jugular venous pressure and positive hepatojugular reflux), and a loud P2 indicative of pulmonary artery hypertension, cor pulmonale would be the most likely diagnosis.

Cor pulmonale refers to hypertrophy or dilatation and/or impaired function of the right ventricle that results from pulmonary hypertension, caused by the diseases of the lung (e.g. chronic obstructive pulmonary disease), vasculature (e.g. idiopathic pulmonary arterial hypertension), upper airway (e.g. obstructive sleep apnea), or chest wall deformities (e.g. kyphoscoliosis)

NOTE - right-sided heart disease due to left-sided heart disease or congenital heart disease is not considered cor pulmonale

Cor pulmonale is the failure of the right heart due to a pulmonary pathology as the underlying
cause. Hypoxia leads to constriction of the pulmonary vasculature and pulmonary hypertension.This eventually results in right heart failure

Many patients may remain asymptomatic for many years. Others may develop acute onset cor pulmonale. Once symptomatic, the clinical manifestation can be as follow:
- Dyspnea on exertion, fatigue, lethargy, and exertional syncope are the results of an inability to sufficiently increase cardiac output during exertion because of increased pulmonary vascular resistance. Dyspnea on exertion generally presents first, followed by fatigue, lethargy, and,eventually, exertional syncope.
- Exertional angina can occur in patients with cor pulmonale, even in the absence of coronary
artery disease. There are two major mechanisms: (1) patients with cor pulmonale have increased myocardial demand; when myocardial demand increases further due to elevated transmural wall tension during exertion, the demand may exceed the supply, precipitating subendocardial right ventricular ischemia. This may be exacerbated by exertional hypoxaemia due to the pulmonary hypertension. (2) angina can be induced during exertion by dynamic compression of the left main coronary artery due to an enlarged pulmonary artery. This, however, do not occuruntil the pulmonary artery trunk diameter exceeds 40 mm.
- Anorexia and right upper quadrant discomfort due to passive congestion of the liver and
bowel (in advanced cases).
- Increased pulmonic component of the second heart sound (may be palpable)
- A narrowly split second heart sound that may be absent if there is a right bundle branch block
- A holosystolic murmur at the left lower sternal border characteristic of tricuspid insufficiency
- A diastolic pulmonary valve regurgitation murmur (in advanced cases)

As the right ventricle becomes hypertrophic, elevated right-sided pressures can produce a
prominent a wave (corresponding to atrial contraction) in the jugular venous pulse, as well as a right-sided fourth heart sound and either a left parasternal heave or a downward subxiphoid thrust. When the right ventricle becomes dilated and fails, the resultant systemic venous hypertension can
produce an elevated jugular venous pressure with a prominent v wave, a right ventricular third heart sound, peripheral edema and, rarely, ascites. The liver can become enlarged and pulsatile, reflecting tricuspid insufficiency. Although peripheral edema is often cited as evidence of cor pulmonale, it is important to keep in mind that peripheral edema can be associated with many of the diseases that
cause cor pulmonale, even in the absence of any hemodynamic abnormalities.

The right-sided murmurs and gallops described above are augmented during inspiration, although this may be obscured by atrial fibrillation or the underlying disease. As examples, the distant heart sounds that are characteristic of COPD and the increased soft tissue of morbidly obese patients who have obstructive sleep apnea make it difficult to identify murmurs and gallops, as well as to
appreciate their augmentation with inspiration.

Patients with end-stage cor pulmonale may develop signs of cardiogenic shock, including
hypotension, tachycardia, oliguria, and cool extremities due to low poor stroke volume. Pulmonary edema may also occur in late-stage disease because elevated right-sided heart pressures can cause the interventricular septum to bow to the left, impairing left ventricular diastolic function.

This patient has signs of advanced right heart failure including edema, raised jugular venous
pressure, and hepatojugular reflux. Since she also has COPD, cor pulmonale is the most likely
diagnosis.

  • http://emedicine.medscape.com/article/154062-overv
  • http://www.merckmanuals.com/professional/cardiovas
39
Q

A 48-year-old man presents to the Emergency Department with complaints of sudden onset chest pain that worsens on respiration and shortness of breath. On physical examination, he has a blood pressure of 110/85 mmHg, a pulse rate of 110 bpm and a respiratory rate of 24 breaths per minute. His jugular vein is distended and a prominent ‘a’ wave of the jugular vein is noted. On heart
auscultation, there is an S4. Which one of the following is the most likely diagnosis?
A. Pulmonary embolism.
B. Infectious endocarditis.
C. Congestive heart failure.
D. Rheumatic fever.
E. Myocardial infarction.

A

A. Pulmonary embolism.

It is rare for pulmonary embolism to instantly lead to acute right heart failure; however,if the
embolism is massive enough, a significantly elevated pulmonary hypertension caused by the massive pulmonaryembolism can result in acute right heart failure which presents with raised jugular venous pressure (JVP), prominent ‘a’ waves, and the emergence of an S4 on auscultationas a result ofan ischemic and/or stiff ventricle.

NOTE - Prominent ‘a’ ways are seen when there is an increased pressure of the right atrium during atrial systole. It can be seen in:
- Tricuspid valve stenosis.
- Diastolic right heart failure.
- Pulmonary hypertension.

(Option B) Infectious endocarditis usually presents differently with fever and murmurs due to valvular involvement. Prominent ‘a’ wave and elevated JVP are not common features of infectious endocarditis.

(Option C) Congestive heart failure is not associated with acute-onset pleuritic chest pain or ‘awave’. However, raised JVP can be noted. An S3 gallop can be present due to volume overload.

NOTE - An S3 is usually normal in children and adults younger than 40 years of age

(Option D) Myocardial infarction (MI) is not associated with pleuritic chest pain. However, MI should always be considered as a differential diagnosis and excluded. A massive MI of right
ventricle with subsequent right hear failure can cause non-pleuritic chest pain, raised JVP and ‘a’
wave.

(Option E) Acute rheumatic fever is characterized by group A streptococcal infection followed by the following clinical manifestations:
Major manifestations:
1. Migratory arthritis (predominantly involving the large joints)
2. Carditis and valvulitis
3. Central nervous system involvement (e.g. Sydenham chorea)
4. Erythema marginatum
5. Subcutaneous nodules
Minor manifestations:
1. Arthralgia
2. Fever
3. Elevated acute phase reactants such as ESR and CRP
4. Prolonged PR interval
To establish the diagnosis of acute rheumatic fever, 2 major manifestations or one major plus two minor manifestations are required. This patient does not meet criteria for acute rheumatic fever.

  • Medscape - Pulmonary Embolism (PE)
  • UpToDate - Overview of acute pulmonary embolism in adults
40
Q

A 37-year-old man presents to the Emergency Department with fever, malaise, and cough for the past 10 hours. He mentions that he has had foursimilar episodes before, but he is quite well in between the episodes. On examination, he has a blood pressure of 130/90 mmHg, pulse rate of 110 bpm, respiratory rate of 24 breaths per minute and temperature of 38.2°C. On chest examination, bilateral crackles are heard. Which one of the following is the most likely diagnosis?
A. Asthma.
B. Asbestosis.
C. Allergic bronchitis.
D. Extrinsic allergic alveolitis.
E. Left ventricular failure.

A

D. Extrinsic allergic alveolitis.

Recurrent episodes of flu like symptoms (fever, malaise, cough) associated with acute shortness of breath are characteristic of extrinsic alveolitis, also known ashypersensitivity pneumonitis. This disease is caused by a hypersensitivity reaction to specific antigens. The subtypes and specific antigens are listed in the following table: See photo below.

Chronic exposure leads to gradual fibrosis of the lungs and bilateral crackles (crepitations).
Removing the offending antigen is the management of choice. Corticosteroids are used to prevent fibrosis, but ineffective if the source of exposure is not removed.

In early stages of the disease, the patient usually does not have interval symptoms. With
progressive fibrosis, exertional dyspnea, pulmonary hypertension, and other characteristic findings of interstitial lung disease develops. In chronic cases, the pulmonary function tests will show intrapulmonary restrictive pattern.

(Option A)Asthma has an intermittent presentation, but the patient does not have fever or malaise.

(Option B) Asbestosis is an occupational lung disease presentingwith progressive shortness of breath, exercise intolerance, and lung fibrosis. There is no intermittent acute exacerbation. Moreover, fever and malaise are not seen in asbestosis.

(Option C) Allergic bronchitis is a variant of asthma in which involvement of bronchi rather than smaller airways are prominent. The main presenting symptom is cough or, less commonly, wheezing. Patients allergic bronchitis are not febrile and do not have malaise during attacks.

(Option E) Although left ventricular failure can result in cough, shortness of breath, and bibasal crepitations, fever and malaise are not expected features. Moreover, the patient is less likely to be asymptomatic in between exacerbations.

  • Medscape - Hypersensitivity pneumonitis
  • Therapeutic Guidelines – Respiratory
40
Q

A 55-year-old man presents to your practice with increasing wheeze and shortness of breath forthe past 2days. His past medical history includes asthma with recurrent episodes with similar symptoms controlledwith oral corticosteroids. His currentmedications areinhaled budesonideand salmetrol. On examination, his blood pressure is 120/80 mmHg, pulse 94 bpm, and respiratory rate 22 breaths per minute. His oxygen saturation is 94% on room air. Respiratory examination is remarkable for widespread wheeze. The rest of the examination is inconclusive. Blood tests shows mild eosinophila and elevated IgE level. Chest X-ray shows central bronchiectasis and patchy opacities. Which one of the following is the most likely diagnosis?
A. Asthma exacerbation.
B. Autoimmune interstitial lung disease.
C. Bronchiectasis.
D. Chronic bronchitis.
E. Allergic bronchopulmonary aspergillosis.

A

E. Allergic bronchopulmonary aspergillosis.

The findings of recurrent asthma exacerbations, patchy opacities and central bronchiectasis on
chest X-ray and eosinophilia and elevated IgE level are suggestive of allergic bronchopulmonary aspergillosis.

The findings of allergic bronchopulmonary aspergillosis are:
- Recurrent episodes of asthma-like attacks
- Migratory pulmonary opacities on chest X-ray
- Central atelectasis on chest X-ray
- Peripheral blood eosinophilia
- Elevated IgE levels
- Positive skin test for aspergillus fugamitus

Treatment is with oral corticosteroids. Inhaled corticosteroids are not effective. Antifungal agents may be added.

  • Medscape - Aspergillosis
  • UpToDate - Allergic bronchopulmonary aspergillosis
41
Q

A 20-year-old man presents to the Emergency Department with complaints of cough, fever, joint pain, and malaise for the past few days. He denies night sweats, weight loss, and chills. He does not smoke. On examination, painful raised erythematosus rashes are observed over his anterior tibia. The rest of the physical exam is inconclusive. A chest X-ray is done that shows bilateral hilar lymphadenopathy. Which one of the following is the most likely diagnosis?
A. Bronchiectasis.
B. Chronic obstructive pulmonary disease (COPD).
C. Tuberculosis.
D. Lung cancer.
E. Sarcoidosis.

A

E. Sarcoidosis.

Fever, malaise, joint pain, cough, and bilateral hilar lymphadenopathy are highly suggestive of
sarcoidosis. The painful raised nodule is consistent with description of erythema nodosum which adds more support to such diagnosis.

Sarcoidosis is a systemic disease of unknown etiology. Histologically, it is characterized by the
presence of non-specific non- caseating granulomas in the lung and other organs. It is more common among black race with a usual age of onset of 20-30 years.

Sarcoidosis commonly involves the lung but may affect any other organ system. There are two distinct sarcoid syndromes with acute presentation:
- Lofgren syndrome: erythema nodosum, arthritis, and hilar lymphadenopathy
- Heerfordt -Waldenstrom syndrome: fever, parotid gland enlargement, uveitis, and facial palsy

(Option A)Bronchiectasis presents with copious sputum production and recurrent chest infections. The scenario does not fit the diagnosis of bronchiectasis.

(Option B) Chronic obstructive pulmonary disease (COPD) can cause cough, but joint pain and rash are not associated features.

(Option C)Tuberculosis (TB) can be anotherdiagnosis. TB can cause cough, fever, and erythema nodosum; however,the history is negative for any risk factors for TB. Moreover, bilateral hilar lymphadenopathy and arthritis make Lofgren syndrome caused by sarcoidosis more likely than TB.

(Option D) Lung cancer is very unlikely in a non-smoker young patient. On the other hand, erythema nodosum, fever, and joint pain are not expected findings in lung cancer.

  • http://emedicine.medscape.com/article/301914-overv
  • Therapeutic Guidelines – Respiratory
42
Q

A 45-year-old non-smoker man comes to your practice with complaint of progressive dyspnea. He has worked in an insulating factory for the last 15 years. Significant physical findings on physical examinationare clubbing, peripheral cyanosis, and inspiratory crackles. A chest X-ray
showsbilateral calcified opacities in both lung fields. For further visualization of the lesions ,you order a high resolution CT scan of the chest. The CT is remarkable for calcified pleural plaques and small opacities in middle lung fields. There is no history of weight loss. Which one of the following is the most likely diagnosis?
A. Pleural effusion.
B. Mesothelioma.
C. Asbestosis.
D. Small cell lung cancer.
E. Squamous cell lung cancer.

A

C. Asbestosis.

The history, clinical findings, and radiological characteristics are consistent with nodular
asbestosis which is a common finding in people withthe history of exposure to asbestos. The
absence of weight loss in history makes the bronchogenic carcinomas a less likely possibility.

Asbestosis is highly associated with both non-malignant and malignant lung disease.

Chest X-ray and spiral CT scan in patients with asbestosis show calcified pleural plaques
suggestive of asbestos exposure.

See photos of Asbestosis on X-Ray & CT scan below.

Pleural mesothelioma (option B) is associated with pleural thickening and calcification, pleuritic chest pain and weight loss, as well as recurrent pleural effusions. There is a connection between asbestos exposure and mesothelioma, but the most common malignancyassociated with asbestosis isbronchogenic carcinoma, not mesothelioma.

Bronchogenic lung cancers are usually seen after 20 years of exposure to asbestosis. However, in the absence of weight loss and lack of supporting findings of a tumor on imaging studies, cancers such as small cell lung cancer (option D) or squamous cell lung cancer (option E) are less likely to be the diagnosis.

Pleural effusion (option A) can cause shortness of breath. With pleural effusion, thelungs will be dullto percussion over the affected areas. It is unlikely for pleural effusion to be missed on both CXR and CT scan.

  • Therapeutic Guidelines - Respiratory
  • Medscape - Asbestosis
43
Q

A 40-year-old woman comes to your clinic with complaint of acute onset shortness of breath. She had one round of chemotherapy 3 weeks ago with both chest exam and X-ray being normal at that time. You assess her and found that her clinical Well score is 4. Which one of the following would be the diagnostic investigation of choice in this patient?

A. D-dimer assay.
B. CTPA.
C. Chest X-ray.
D. Doppler ultrasound scan.
E. V/Q scan.

A

E. V/Q scan.

Not all patients presenting with possible symptoms of pulmonary embolism (PE) need to undergo imaging tests such as computer tomography pulmonary angiography (CTPA), V/Q scan, Doppler ultrasound, etc. Decision as to whether this tests are performed depends on pretest probability of PE. A reasonable way to stratify patient risk is to use one of the validated clinical decision rules (CDR). simplified Wells Score (SWS) is one the most commonly used CDR for this purpose. It takes into account different variables to generate a score, based on which the probability of PE will be estimated to be low, moderate or high. The variables and their designated score are according to the following table: - See table below.

Clinical probability of PE based on Wells score: - See table below.

Or

  • Wells score ≤ 4: PE unlikely
  • Wells score >4: PE likely

Those with low or intermediate pretest probability should have a D-dimer assay as the most appropriate step. D-dimer, a degradation product of cross-linked fibrin, is elevated in the presence of clot because of the activation of fibrinolysis. A negative D-dimer using a quantitative enzyme- linked immunoabsorbent assay (ELISA) has a sensitivity of >95% and effectively excludes PE in low- and intermediate-probability groups. However, D-dimer levels are elevated after surgery, in cancer patients, and during pregnancy and cut-off levels considered for other patients may not be as reliable. In the presence of cancer D-dimer is not a useful tests and imaging studies should be considered.

CTPA and V/Q scan are used to confirm the diagnosis of PE. Diagnostic accuracy of CTPA and V/Q scan is similar. However, CTPA detects clots in smaller vessels. CTPA may have the advantage of widespread availability whereas V/Q scanning may not be available outside working hours. Radiation dose of V/Q is significantly less than CTPA, making V/Q scan preferable for women of reproductive age (< 55 years) to avoid the risk of breast cancer associated with higher radiation exposure in CTPA, Provided that the patient has a clear chest X-ray and no underlying lung pathology to make V/Q scan results equivocal.

NOTE - Although CXR is the initial test of choice for every patient with suspected PE, it is only used to exclude other causes of such presentation as well as eligibilty for V/Q scan. It is not diagnostic for PE. However, V/Q or CTPA are not used as first tests for every patient with suspected PE, unless there is a high index of suspicion based on pre-test probability.

44
Q

You are called to see a 60-year-old man presenting with dyspnea and chest pain that worsens on inspiration. Five days ago, he underwent laparotomy and gastric surgery. On physical examination, he has a temperature of 37.5°C, heart rate of 90 bpm and blood pressure of 130/95 mmHg. Chest auscultation reveals no abnormal heart or respiratory sounds. There is good air entry into both lungs and the percussion note is resonant in all areas. A chest X-ray and a ventilation/perfusion scan are performed. Which one of the following test results indicates a high probability of pulmonary embolus in a particular zone of the lung?

A. Normal chest X-ray, abnormal ventilation, normal perfusion scan.
B. Normal chest X-ray, normal ventilation, reduced perfusion.
C. Consolidation on chest X-ray, normal ventilation, reduced perfusion.
D. Consolidation on chest X-ray, abnormal ventilation scan, normal perfusion scan.
E. Normal chest X-ray, abnormal ventilation scan, reduced perfusion scan.

A

B. Normal chest X-ray, normal ventilation, reduced perfusion.

Chest radiographs are the first imaging modality in assessment of a patient with suspected pulmonary embolism (PE) to exclude other pulmonary causes of the patient’s presentation.

Initially, chest X-rays are normal in most cases of PE; however, in later stages, most patients
will develop chest X-ray abnormalities including, atelectasis, small pleural effusions, parenchymal opacities and elevation of the hemidiaphragms.

Once pulmonary infarction occurs, the classic radiographic finding is a wedge-shaped, pleural- based triangular opacity with an apex pointing towards the hilus (Hampton hump). This finding is neither common, nor specific. Another infrequent and non-specific finding is decreased pulmonary vascularity and dilated pulmonary vessels (Westermark sign). In general chest X-rays cannot be used to include or exclude PE; however, radiography and ECG may be useful for excluding alternative diagnoses.

Ventilation/perfusion scan (V/Q scan) or radioisotope lung scanning is performed using injecting of particles of albumin labeled with technetium-99m. As these particles perfuse the lung, the lungs are imaged by using a gamma camera to obtain anterior, posterior, lateral, and oblique views. In normal lungs, the isotope particles distribute evenly and produce two dark lung-shaped shadows. In PE, the embolus blocks the branches of the pulmonary artery producing a filling defect on the scan. When findings in the perfusion scan are abnormal, a ventilation scan is obtained by using inhalational radioactive xenon-133. Uncomplicated PE does not make any alteration in ventilation scan. As a result a patient with a high probability of PE has an abnormal perfusion scan with a normal ventilation component.

Any pre-existing lung abnormality such as COPD, pneumonia, atelectasis, etc, by altering the ventilation scan results, can make interpretation of a V/Q scan difficult and inconclusive; hence, a clear chest X-ray is the essential prerequisite for performing a V/Q scan.

With PE, the findings on V/Q scan of this patient at early stages of PE will be a normal chest X-ray, normal ventilation scan, and reduced perfusion on perfusion scan.

45
Q

A 34-year-old man presents to the Emergency Department with acute-onset shortness of breath. Medical history is only remarkable for essential thrombocytosis. On examination, he has a blood pressure of 110/70 mmHg, pulse rate of 90 bpm, respiratory rate of 22 breath per minute, and temperature of 37.1°C. He has a BMI of 24 kg/m2. Chest radiograph is significant for a prominent right hilum. With the provisional diagnosis of pulmonary embolism, a CT pulmonary angiogram (CTPA) is ordered which is shown in the following photograph. Which one of the following is the most appropriate treatment option for this patient?

A. Warfarin.
B. Unfractionated heparin or low-molecular weight heparin (LMWH).
C. Caval filter.
D. Thrombolytic therapy.
E. Dabigatran.

A

B. Unfractionated heparin or low-molecular weight heparin (LMWH).

The photograph shows a filling defect in the right branch of the pulmonary artery consistent with the diagnosis of pulmonary embolism (PE).

PE may only present with shortness of breath and tachypnea. However, other symptoms and signs such as pleuretic chest pain, cough, tachycardia and, in severe cases, hemodynamic instability may be present.

With confirmed PE, the patient should be started on unfractionated heparin or low-molecular weight heparin (LMWH) as the most appropriate next step in management.

Option A Warfarin should be commenced within 48 hours of heparin administration and continued for at least 3 months or more, depending on individual risk of recurrent PE or DVT in the future.

Option C Inferior vena cava (IVC) filters provide a screen in the inferior vena cava, allowing blood to pass through, while large emboli from the pelvis or lower extremities are blocked or fragmented before reaching the lung. Placement of an IVC filter is generally considered in patients who have contraindications to anticoagulation, failed anticoagulation, or developed a complication due to anticoagulation, or are non-adherent to treatment. Other indication is hemodynamic or respiratory compromise significant enough to make another PE may be fatal.

Option D Thrombolytic therapy may be considered for patients with PE associated with significant hemodynamic compromise. This patient does not have hemodynamic instabiliy justifying the use of thrombolytics.

Option E Dabigatran is a direct thrombin inhibitor, approved in Australia for stroke prevention in patients with non-valvular atrial fibrillation and at least one other risk factor for stroke. It is not recommended to substitute warfarin or heparin therapy in PE.

46
Q

A 52-year-old woman presents to your GP clinic with severe upper arm pain and progressive shortness of breath for the past 6 months. She is none-smoker with a medical history significant only for an operation on her right thigh approximately 2 years ago for a strangulated right femoral hernia. On examination, weakness and muscle wasting of the right hand is noted, as well as an increased respiratory rate of 20 breaths per minute. A chest X-ray is obtained that is shown in the following photograph. Which one of the following would be the most appropriate next step in management?

A. Lung biopsy.
B. Electromyographic studies of the right arm.
C. Electromyographic studies of the right thigh.
D. Bronchoscopy.
E. Sputum cytology exam.

A

A. Lung biopsy.

The photograph shows a cap-like opacity covering the apex of the right lung. This finding in addition to hand weakness and muscular atrophy and upper arm pain is suggestive of Pancoast tumor as the most likely diagnosis.

The terms “Pancoast tumors”, “superior sulcus tumors”, or “superior pulmonary sulcus tumors” are used to describe neoplasms located at the apical pleuropulmonary groove, adjacent to the subclavian vessels.

Lesions in the superior sulcus can result in shoulder or arm pain in the distribution of the C8, T1, and T2 dermatomes. T1 involvement causes sensory disturbances in the medial forearm and hand and wasting of intrinsic hand muscles.

Horner’s syndrome can be another finding. It is caused by invasion to the stellate sympathetic ganglion in the neck. The constellation of Horner’s syndrome and weakness and atrophy of hand muscles is referred to as “Pancoast’s syndrome”.

The majority of patients with superior sulcus tumors present with one or more of the following:

Shoulder pain - The most common initial symptom of superior sulcus tumors is shoulder pain, present in 44%-96% of patients. Pain is produced by invasion to the brachial
plexus and/or extension of the tumor into the parietal pleura, endothoracic fascia, first and second ribs, or vertebral bodies. Pain can progress and radiate up to the head and neck, or down to the medial aspect of the scapula, axilla, anterior chest, or ipsilateral arm in the distribution of the ulnar nerve. Patients frequently receive treatment for presumed cervical osteoarthritis or shoulder bursitis and there is often a delay in diagnosis. Pain ultimately becomes persistent and often so severe that the patient may need to support the involved arm with the uninvolved hand for pain relief.

Horner’s syndrome - Horner’s syndrome consists of ipsilateral ptosis with narrowing of the palpebral fissure, miosis, enophthalmos, and anhidrosis. It is caused by the involvement of the paravertebral sympathetic chain and the inferior cervical (stellate) ganglion. It is seen in 14%-50%

Ipsilateral flushing and decreased sweating of the face may occur before the development of the full Horner’s syndrome, presumably due to irritation of the sympathetic chain by the tumor prior to frank invasion. Ipsilateral complex regional pain syndrome (reflex sympathetic dystrophy) and facial pain similar to paroxysmal hemicrania rarely may occur. Contralateral facial sweating and flushing, especially with exercise, due to an excessive response by the intact sympathetic pathway have also been reported; this phenomenon is known as the Harlequin sign.

Neurologic complications involving the upper extremity - Extension of the tumor to the C8 and T1 nerve roots results in upper extremity neurologic findings in approximately 8%-22% of cases. Involvement of these nerve roots may result in weakness and atrophy of the intrinsic hand muscles, or pain and paresthesia of the fourth and fifth digits and the medial aspect of the arm and forearm. Abnormal sensation and pain in the T2 territory (the axilla and medial aspect of the upper arm) may also be an early finding. The triceps reflex may be lost.

Superior sulcus tumors invade the intervertebral foramina early in the course of disease in approximately 5% of patients and may cause spinal cord compression and paraplegia. Approximately 25% of patients ultimately develop spinal cord compromise.

Other findings - Supraclavicular lymph node enlargement and prominent weight loss are seen in some patients. Superior sulcus tumors may produce a phrenic or recurrent laryngeal neuropathy or superior vena cava syndrome in 5%-10% of patients.

Radiologic findings include a unilateral cervical cap of more than 5 mm, asymmetry of bilateral cervical cap of more than 5 mm, an apical mass and local bone destruction.

Biopsy is used for definite diagnosis. The location of the superior sulcus tumors allows percutaneous needle biopsy in most cases. The technique can be performed via a posterior or cervical approach with the use of ultrasound or CT to localize the lesion. Video-assisted thoracoscopy (VATS) or thoracotomy can be performed if less invasive methods are non- diagnostic.

CT scanning of the chest is often used to provide additional information about a superior sulcus tumor and its extension, including the presence of satellite pulmonary nodules, parenchymal disease, and mediastinal lymphadenopathy. MRI is a better option for evaluation of brachial plexus, subclavian vein and artery and chest wall involvement. CT scan and MRI are often performed prior to biopsy and were the best choices if present among the given options. In their absence, lung biopsy would be the most appropriate next step and the correct option.

(Options B and C) Electromyographic studies do not add any useful infrmation regarding diagnosis or management plan.

(Options D and E) Due to the peripheral location of superior sulcus tumors, the diagnostic yields of bronchoscopy and sputum cytology are less desirable. Fiberoptic bronchoscopy with cytology and biopsies are diagnostic in only 30% to 40% of patients, but there is the benefit of detecting unexpected endobronchial tumors that will significantly change the management plan.

47
Q

A 45-year-old man presents with sudden onset severe chest pain and shortness of breath 2 weeks after a hip replacement surgery. One year ago, he was diagnosed with chronic renal disease due to IgA nephropathy. Which one of the following is the most appropriate investigation to consider for this patient?

A. Computed tomography pulmonary angiography (CTPA). B. V/Q scan.
C. ECG.
D. Chest X-ray (CXR).
E. D-dimer.

A

B. V/Q scan.

Hip or knee surgery is a strong risk factor for venous thromboembolism i.e. deep vein thrombosis (DVT) and pulmonary embolism (PE). In patients with risk factors for PE, strong risk factors in particular, any suggestive sign and symptoms should prompt detailed investigations. V/Q scan and CTPA are the diagnostic methods to confirm the diagnosis of PE in whom, based on their pretest probability test, PE is considered likely.

Generally, CTPA is preferred over V/Q scan because it is often readily available, easy to use and more convenient to the patient. Unlike V/Q scan, underlying lung disease evident on CXR does not significantly affect the specificity of the tests. Besides, it can detect smaller and more distal emboli in pulmonary arterial tree, as well as other conditions that may mimic PE. Drawbacks to this method are more radiation exposure and detection of clinically insignificant emboli which can trigger unnecessary anticoagulation.

CTPA is always the diagnostic method of choice except in:

  • Women of reproductive age (<55 years)
  • Those with poor GFR (<40) or those with progressively declining renal function
  • Those with known allergic reaction to contrast media

This patient has poor GFR due to chronic kidney disease. In patients with poor GFR or those with rapidly declining renal function, contrast media used in CTPA is relatively contraindicated and should be avoided where possible. Contrast media can result in more kidney damage by (1) direct toxic effect on kidney tubules and (2) triggering allergic reactions resulting in allergic interstitial nephritis, both of which have detrimental effects on a previously damaged kidney. Having said these, V/Q scan should be considered in this patient for definite diagnosis and the most appropriate step.

ECG and CXR should be performed for all patients with suspected PE to exclude other mimickers, but they are not diagnostic whatsoever.

D-dimer assay is used to exclude PE in patients in whom PE is unlikely based on pretest probability assessment with one of validated clinical decision rules (CDR) such as Wells score, revised Geneva score, etc.

NOTE Patients with contraindication to contrast material, such as this patient, are excluded from most CDRs.

48
Q

An inpatient 65-year-old man develops acute onset-chest pain and dyspnea 7 days after hemicolectomy due to colon cancer. On examination, his BP is 140/95 mmHg, heart rate 106 bpm and respiratory rate 22 breaths per minute. Which one of the following is the most appropriate next step in management of this patient?

A. Computed tomography pulmonary angiogram (CTPA).
B. Chest X-ray (CXR).
C. ECG.
D. D-dimer.
E. V/Q scan.

A

C. ECG.

Sudden onset dyspnea with or without other signs and symptoms of PE such as pleuretic chest pain, persistent tachycardia, hypotension, cough, etc. in a background of recent surgery and the absence of a reasonable alternative explanation should raise the suspicion of PE. In general, individual clinical signs and symptoms do not allow exclusion or confirmation of PE. However, the history of recent major surgery increases the probability of this diagnosis.

While PE is a possible diagnosis, other conditions that can mimic PE should always be kept in mind and excluded first. Although pleuretic chest pain goes against acute coronary syndrome (ACS) as a likely diagnosis, in the presence of dyspnea, especially in the post-operative period, performing an ECG is mandatory to exclude cardiac ischemia both as a cause of the chest pain, a concomitant event, or as a result of hypoxia and cardiac ischemia triggered by the suspected PE.

By ECG, ACS as a potential cause to the presentation, can be excluded and possible concomitant abnormalities be assessed for risk stratification.

The most common ECG abnormalities in the setting of PE are tachycardia and non-specific ST-T wave abnormalities. The classic findings of right heart strain and acute cor pulmonale caused by PE, as an underlying cause, are tall peaked P waves in lead II (P pulmonale), right axis deviation, right bundle-branch block, an S1Q3T3 pattern, or arterial fibrillation. These findings, however, are present in only about 20% of patients with PE. If they are present, they can only be suggestive (not diagnostic), but their absence does not exclude PE whatsoever.

A chest X-ray comes next in line to not only exclude other caused to the presentation, but also to decide upon the imaging modality of choice for definite diagnosis of PE.

(Option A) CTPA (or V/Q scan) was the answer if the most appropriate diagnostic test was asked. Generally CTPA is preferred over V/Q scan as it is faster to perform and more available. Furthermore, it can detect other pulmonary abnormalities other than PE, or where an abnormal chest X-ray precludes use of V/Q scan due to potential non-diagnostic results.

(Option B) In general, ECG and chest-x ray are always among initial steps to exclude PE not to diagnose it. After an ECG, chest X-ray is the most appropriate next step in management.

(Option D) D-dimer assay is a useful screening tool for patients in whom PE is considered ‘unlikely’ based on clinical decision rules (CDRs) such as Wells score, simplified Wells score, etc.

(Option E) V/Q scan is the most appropriate diagnostic modality in pregnant patients, in those with allergy to contrast media, in patients with decreased GFR due to renal disease and in women of reproductive age due to increased radiation exposure and potential risk of breast cancer associated with CTPA.

49
Q

A 42-year-old woman sustained a motor vehicle accident (MVA) and had her femoral neck fractured which is surgically fixed. Five days after the surgery, she develops sudden-onset chest pain and dyspnea. She is on oral contraceptive pills. On examination, she is afebrile, has a heart rate of 100 bpm, BP of 110/70 mmHg, and respiratory rate of 24 breaths per minute. Lungs are clear to auscultation. Chest X-ray and ECG are normal. Which one of the following is the most appropriate next step in management?

A. Duplex doppler ultrasound scan of lower limbs.
B. D-dimer assay.
C. Computed tomography pulmonary angiography (CTPA). D. Arterial blood gas (ABG) analysis.
E. V/Q scan.

A

E. V/Q scan.

Orthopedic surgery is a strong risk factor for development of venous thromboembolism in the post- op period. For every patient with major surgery within the past 4 weeks, or immobilization longer than 3 days, who have signs and symptoms such as dyspnea, chest pain (especially pleuretic or sudden onset), cough, hemoptysis, or hypoxia, pulmonary embolism (PE) should always be suspected and promptly investigated.

CTPA and V/Q scan are confirmatory tests for PE if, through pretest probability assessment, PE is a likely diagnosis. There are rules referred to as Clinical Decision Rules (CDR) for this purpose. CDRs take into account different items to calculate the likelihood of PE as a diagnosis to
prevent unnecessary diagnostic testing or treatment.

One of the most recently validated and practical of these CDRs is Simplified Wells Score (SWS). SWS gives seven items one score each (if present). Patients with scores 2 or higher are likely to have PE and should undergo imaging studies either by CTPA or V/Q scan, while patients with scores 0 or 1 are unlikely to have PE. In this group, a D-dimer assay is used to safely exclude PE.

D-dimer is a fibrin degeneration product. High levels of D-dimer represent the presence of thrombus and triggers further confirmatory diagnostic tests for definite diagnosis. Negative results have high negative predictive value and are sensitive enough to exclude PE in patients with low to moderate pretest probability of PE. D-dimer levels are, however, elevated in post-surgery (first 2 weeks) period, pregnancy, inflammation, infection, and cancer, making a negative result less likely and the test less desirable where these conditions are present.

The following diagram shows the approach to patients with suspected PE bases on SWS: - See diagram below.

This patient has had a major surgery within the past four weeks (1 score). She also has a clear chest X-ray and a normal ECG, excluding other lung pathologies and cardiac problems as a cause to the presentation; hence, no other condition seems to be more likely (1 score). She has a SWS 2 and is likely to have PE.

When PE is likely, imaging tests follow. Generally, CTPA is preferred over V/Q scan because it is often readily available, easy to use and more convenient to the patient. Unlike V/Q scan, underlying lung disease evident on CXR does not significantly affect the specificity of the test. Moreover, it can detect smaller and more distal emboli in pulmonary arterial tree, as well as other conditions that may mimic PE. Drawbacks to this method are more radiation exposure and detection of clinically insignificant emboli which can trigger unnecessary anticoagulation.

CTPA is always the method of choice except in the following groups:

  • Women of reproductive age (<55 years)
  • Those with poor GFR (<40) or those with progressively declining renal function
  • Those with known allergic reaction to contrast media

Since this patient is a female younger than 55 years, V/Q is scan is the preferred method, as radiation exposure is greater with CTPA and associated with higher rates of breast cancer in future, at least theoretically.

When leg symptoms are present (a minimum of leg swelling or tenderness of deep veins on palpation), a Doppler ultrasonography of the lower limbs can be used as an alternative. The usual source of PE is thrombosis in the deep veins (DVT) of the lower limbs or pelvis. If a clot is present in lower limb or pelvis on Doppler ultrasound, no further imaging is required, and treatment should be started. In the absence of leg symptoms, however, Doppler ultrasound is unlikely to be useful.

NOTE - Although ABG is an essential part of PE workup alongside the ECG and CXR, it is not the correct answer here because even with a normal ABG or an ABG inconsistent with expected result in PE (hypoxia and hypocarbia at the same time) is neither sensitive nor specific. ABG is never used to diagnose PE with certainty.

50
Q

A 28-year-old man sustains multiple limb fractures in a motor vehicle accident and undergoes surgical fixation of the fractures. On the third day after the surgery, he develops sudden right-sided pleuretic chest pain and shortness of breath. On examination, the findings are otherwise inconclusive. Which one of the following would be the most likely diagnosis?

A. Pulmonary embolism.
B. Fat embolism.
C. Atelectasis.
D. Pneumonia.
E. Air embolism.

A

A. Pulmonary embolism.

The history of orthopedic surgery and clinical findings of pleuritic chest pain and shortness of breath make pulmonary embolism (PE) the most likely diagnosis. PE may have been caused by clot (thromboembolism) or fat from a fractured bone where fat can escape from bone marrow into circulation.

NOTE -
differential diagnoses of pleuritic chest pain include the following 5 ‘P’s:
1. Pulmonary embolism
2. Pericarditis
3. Pneumonia
4. Pneumothorax
5. Pneumomediastinum.

(Option B) Fat embolism syndrome (FES) is systemic involvement due to occlusion of vessels by free-circulating fat drops and droplets. Brain, eyes, kidneys, skin, and lungs are the most common affected organs. Lung involvement is termed as pulmonary embolism, not fat embolism because the latter is, by definition, a systemic condition not just confined to the lungs.

The history of a patient with fat embolism syndrome (FES) may include the following:

  • Trauma to a long bone or the pelvis, including trauma sustained in orthopedic procedures
  • Parenteral lipid infusion
  • Recent corticosteroid administration

Major physical criteria of FES are respiratory symptoms, cerebral signs without other etiologies and petechial rash. Minor criteria are pulse rate higher than 110 bpm, fever higher than 38.5° C, retinal changes of fat globules or petechiae, renal dysfunction, jaundice, acute drop in hemoglobin level, platelet count, or both and elevated erythrocyte sedimentation rate
A formal diagnosis of FES requires the presence of at least one major criterion and four minor criteria, plus fat microglobulinemia. With only shortness of breath and pleuretic chest pain and otherwise normal findings on exam this patient is unlikely to have FES.

(Options C and D) Atelectasis after the first day of operation, and pneumonia without preceding atelectasis are less likely.

(Option E) Air embolism is a rapidly lethal condition with sudden death being the most common presentation. It does not cause pleuritic chest pain. Often the patient will not have enough time to even become short of breath.

51
Q

A 20-year-old man is admitted after a motor vehicle crash, in which he sustained multiple lower limb fractures. He underwent surgical fixations of his fractures. Five days after the surgery, he becomes pale, clammy and short of breath. His BP is 110/70 mmHg and pulse rate 100 bpm. Which one of the following investigations is most likely to diagnose his current problem?

A. Duplex Doppler ultrasound of the calf veins.
B. Computed tomography pulmonary angiography (CTPA). C. D-dimer assay.
D. Positron emission tomogram (PET).
E. Chest X-ray (CXR).

A

B. Computed tomography pulmonary angiography (CTPA).

Orthopedic surgery and immobilization (bed rest) more than 3 days are strong risk factors for development of venous thromboembolism during the post-op period. For every patient with risk factors for PE, who have signs and symptoms such as dyspnea, chest pain (especially pleuretic or sudden onset), cough, hemoptysis, or hypoxia, pulmonary embolism should always be suspected and promptly investigated.

CTPA and V/Q scan are confirmatory tests for definite diagnosis of PE. Generally, CTPA is preferred over V/Q scan because it is often readily available, easy to use and more convenient to the patient. CTPA also comes first when the patient appears to be hemodynamically unstable. Although this patient has normal blood pressure and heart rate, his pallor and clamminess are ominous signs heralding shock. He requires the fastest method to establish the diagnosis prior to starting treatment.

CXR (option E), ECG, and ABG are all performed on patients with suspected PE but none of them are diagnostic for it.

D-dimer assay **(option C) **is not a reliable tool in post-op patient as it is usually high after surgery and has no positive predictive value for PE in such patients. Other conditions where D-dimer assay is unreliable are during pregnancy and shortly in postpartum period and in patients with malignancies.

Doppler ultrasound (option A) could be diagnostic in patients with suspected PE and simultaneous leg symptoms suggestive of deep vein thrombosis (DVT), sparing the patient the need to undergo CTPA or VQ scanning. However, in the absence of such symptoms, doppler ultrasound is very unlikely to be diagnostic.

Positron-emission tomogram (PET) (option D)is of little diagnostic value in suspected PE and nor recommended.

NOTE - CTPA is always the method of choice except in the following groups:

  • Women of reproductive age (<55 years)
  • Those with poor GFR (<40) or those with progressively declining renal function
  • Those with known allergic reaction to contrast media
52
Q

A 62-year-old man presents to the Emergency Department with sudden-onset chest pain and shortness of breath. He has chronic kidney disease and a documented reduced GFR of 25 ml/min. On examination, his blood pressure is 110/75mHg and pulse 112 bpm. An ECG is performed that is is normal. A chest X-ray shows a wedge-shaped opacity in the periphery of the right lung. Which one of the following should be considered the best diagnostic tool in this patient?

A. Duplex Doppler ultrasound of lower limbs.
B. D-dimer assay.
C. Computed tomography pulmonary angiography (CTPA). D. Arterial blood gas (ABG) analysis.
E. V/Q scan.

A

E. V/Q scan.

Sudden onset of shortness of breath can have a variety of etiologies. Spontaneous pneumothorax and pulmonary embolism (PE) are typical examples. Pneumonia and other respiratory infections are also possible explanations, as is cardiac ischemia. Pericarditis and pneumomediastinum are also possible.

In approach to such patients, an ECG and chest radiograph are the initial steps to narrow down the possibilities. An arterial blood gas (ABG) analysis is indicated as well in the presence of hypoxia.

There is no mention of fever in the question; therefore, infectious causes are weak possibilities. ECG is normal and excludes cardiac ischemia as a cause.

A wedge-shaped opacity on CXR, however, is suggestive of pulmonary infarction, making pulmonary embolism likely.

With this amount of information, PE comes first on the differentials list. In other words, no other etiology could be as likely as PE. At least PE can be as possible as other causes of this presentation.

Definite diagnosis of PE relies on imaging studies such as CTPA or V/Q scan; hiowever, not everyone with signs and symptoms of PE should undergo diagnostic imaging just based on the presentation. To avoid unnecessary imaging studies on the one hand and missing patients with PE as a potential fatal condition on the other, clinical decision rules (CDRs) are devised to help clinicians identify patients who are likely or unlikely to have PE and to avoid performing imaging in those who have low or intermediate risk.

There are a number of validated CDRs for clinical use with Wells and revised Geneva being most commonly applied in clinical situations; however, with either of the following conditions present, patients with suspected PE should be excluded from assessment with Wells score, simplified Wells score, and revised Geneva score:

  • Ongoing anticoagulant treatment
  • Contraindication to CT (known allergy to iodinated contrast agents or risk for allergic reaction, creatinine clearance (GFR) <0.5ml/s or <30ml/minute)
  • Pregnancy
  • Suspected massive PE with shock
  • Estimated life expectancy less than 3 months

This patient, who has chronic kidney disease and poor GFR, is not an appropriate candidate for pretest probability assessment using Wells score, either classic or simplified, or revised Geneva score, and should be considered for investigation with imaging studies. Moreover, with PE being more likely than other possible conditions (1 score) and tachycardia (1 score), he has a simplified Wells score of 2 and should have imaging tests as the next best step.

Generally, CTPA is preferred over V/Q scan because it is often readily available, easy to use and more convenient to the patient. Unlike V/Q scan, underlying lung disease evident on CXR, does not significantly affect the specificity of the test. Moreover, it can detect smaller and more distal emboli in pulmonary arterial tree, as well as other conditions that may mimic PE. Drawbacks to this method are more radiation exposure and detection of clinically insignificant emboli which can trigger unnecessary anticoagulation.

CTPA is always the method of choice except in the following patient groups:

  • Women of reproductive age (<55 years)
  • Those with poor GFR (<40) or those with progressively declining renal function
  • Those with known allergic reaction to contrast media

This patient has poor GFR due to chronic kidney disease. In patients with poor GFR or those with rapidly declining renal function, contrast media used in CTPA is relatively contraindicated and should be avoided where possible. Contrast media can result in more kidney damage by (1) direct toxic effect on kidney tubules and (2) triggering allergic reactions resulting in allergic interstitial nephritis. For such patients, V/Q scan should be considered for definite diagnosis and as the most appropriate step. CXR abnormalities (the wedged-shape opacity here) makes V/Q scan less ideal because the chances are that the result is misinterpreted, yet V/Q scan is the only alternative given the risk of contrast media to this patient.

53
Q

A 26-year-old woman presents to the emergency department with shortness of breath and pleuretic chest pain. Based on the history and clinical findings, she is suspected to have pulmonary embolism. She has a family history of thrombophilia. Which one of the following is the investigation of choice for her?

A. Duplex Doppler ultrasound scan of lower limbs.
B. Chest X-ray (CXR).
C. V/Q scan.
D. Computed tomogram pulmonary angiogram (CTPA).
E. D-dimer assay.

A

C. V/Q scan.

Computed tomography pulmonary angiography (CTPA) and V/Q scan are currently imaging modalities used for definite diagnosis of pulmonary embolism (PE).

Generally, CTPA is preferred over V/Q scan because it is often readily available, easy to use, and more convenient to the patient. Unlike V/Q scan, underlying lung disease evident on chest X-ray does not affect the specificity of the test. Besides, it can detect smaller and more distal emboli in pulmonary arterial tree, as well as other conditions that may mimic PE. Drawbacks to this method are more radiation exposure and detection of clinically insignificant emboli which can trigger unnecessary anticoagulation.

CTPA is always the method of choice except in the following patient group:

  • Women of reproductive age (<55 years)
  • Those with poor GFR (<40) or those with progressively declining renal function
  • Those with known allergic reaction to contrast media
  • Pregnant women

This woman is young (<55 years) and V/Q scan is the preferred diagnostic option for her. It is recommended that V/Q scan be used in women of reproductive age (<55 years) to avoid the increased risk of breast cancer associated with higher radiation exposure in CTPA.

(Option A) Doppler ultrasound is used for detection of lower limb clot(s) as the source of PE. Lower limb clots indirectly point towards PE as the most likely cause to the presentation. In the absence of leg symptoms, however, this method does not yield high diagnostic values.

(Option B) CXR is performed in all patients with suspected PE to exclude other possible causes and guide as to selection of the imaging modality of choice. V/Q scan is less accurate when there are chest X-ray abnormalities. CXR is not a diagnostic test for PE ecause it is neither sensitive, nor specific.

(Option D) CTPA is better avoided in women younger than 55 years due to increased risk of breast cancer associated with higher radioation exposure compared to V/Q scan.

(Option E) D-dimer assay is indicated as a screening tool in patients with low or intermediate pretest probability of PE estimated with current clinical decision rules (CDR). It is not diagnostic.

54
Q

A 40-year-old woman presents to the Emergency Department with sudden-onset chest pain, dyspnea and a fever of 39°C. She has arrived from Canada to Sydney 2 days ago by air. Which one of the following would be the most appropriate next step in management?

A. Chest X-ray (CXR).
B. ABG.
C. Computed tomography pulmonary angiography (CTPA). D. V/Q scan.
E. D-dimer assay.

A

A. Chest X-ray (CXR).

Travel, especially by air, longer than 3 hours is a risk factor for development of thromboembolism either in form of deep vein thrombosis (DVT) or pulmonary embolism (PE). Sudden onset dyspnea is the most common presenting symptom in PE. Pleuretic chest pain, cough, hemoptysis, hypoxia, and even collapse and hemodynamic instability can be present as well. Fever is less common, yet a possible presenting sign. However, in the presence of fever, more common causes should be excluded before PE is considered as a likely diagnosis. A CXR is always required as one of the very initial measure to investigate other likely causes other than PE. This is even more important in the presence of fever that makes an infectious etiology more likely than or at least as likely as, PE.

(Option B) ABG is performed in initial assessment. Typical finding for PE is concomitant presence of hypoxia and hypocapnia; however, this pattern is seen in only a minority of patients. Hypoxia is frequently present but may be due to a wide variety of other cardiopulmonary complications. To put it simple, ABG does not add diagnostic value to this case.

(Options C and D) CTPA and V/Q scan are used for definite diagnosis of PE if initial assessment favors it but not as the next best step here.

(Option E) D-dimer assay is used when PE is considered as a likely diagnosis in patients with low to moderated pretest probability for PE, after other potential causes are excluded.

55
Q

A 6-year-old boy is brought to your attention because of frequent episodes of wheezing and cough at night and during exercise. You prescribe salbutamol on as-needed basis and regular low-dose inhaled fluticasone. Which one of the following medications is most effective in preventing further episodes?

A. Long-acting beta agonists (LABA). B. Short-acting beta agonists (SABA).
C. Sodium cromoglycate (SCG).
D. Fluticasone.
E. Montelukast.

A

D. Fluticasone.

This child has persistent asthma evident by frequent episodes of night symptoms more than 2 nights. The scenario does not provide adequate information as to whether this is moderate or severe persistent asthma.

Asthma severity classification based on frequency of symptoms is as follows:

Intermittent
* Symptoms occur more than one week apart AND
* Night symptoms occur less than 2 time per month (1 night per month only or not at all)

Mild persistent
* Symptoms occur more than once a week but not on a daily basis
* Night symptoms occur more than 2 times per month

Moderate persistent
* Symptoms occur on a daily basis
* Night symptoms more than once in a week

Severe persistent
* Symptoms occur on a daily basis Frequent night symptoms

For persistent asthma, the treatment of choice is regular use of an inhaled corticosteroid (ICS) and a short acting beta agonist (SABA) on a as-needed basis. Treatment should be started at the lowest dose possible and titrated according to the response.

ICS (fluticasone, beclomethasone, budesonide, ciclesonide) are the most effective preventive therapy in children with asthma. In Australia, they are referred to as ‘preventers’ and are first-line maintenance treatment.
This child has already been started on inhaled fluticasone. None of the options are superior to fluticasone as a preventive measure.

(Option A)
LABAs are not routinely used as preventers.

(Option B) SABA is the treatment of choice for intermittent asthma, or for prevention of wheeze and cough that is only induced by exercise.

(Option C) Cromones (cromoglycate, nedocromil) can be used as an alternative to low-dose ICS or montelukast in children with frequent intermittent or persistent asthma, but they are not as effective of ICS and require a more complex regimen. They should be used more frequently (e.g. 4 times a day) and meticulous daily care needed care to prevent clogging of the inhaler device. Nedocromil has an unusual unpleasant taste and is not tolerated by many children.

(Option E) Although not as effective as ICS, a 2-week trial of montelukast maybe considered first in an attempt to spare the child of ICSs.

56
Q

A 12 year-old boy is brought to the emergency department by his mother with complaints of cough and wheezing. He has had previous episodes in the past, and has not any specific treatment so far. On examination, he has a respiratory rate of 40 breaths per minute and a temperature of 37.5°C. On chest auscultation, widespread wheeze is noted bilaterally. The rest of the exam is inconclusive. Which one of the following is most likely to make a diagnosis now and in the emergency setting?

A. CT scan of the chest.
B. Chest X-ray (CXR).
C. Spirometry.
D. Improvement with salbutamol.
E. Measurement of peak expiratory flow (PEF).

A

D. Improvement with salbutamol.

The scenario is typical for an exacerbation of a previously-undiagnosed asthma. Asthma is the most common cause cough and wheezing in children.

By definition, asthma is reversible inflammatory response of the bronchial tree triggered by factors such as allergens, cold, exercise, stress, noxious inhalants, dust mite, etc. Reversibility and symptom relief after administration of bronchodilators (e.g. inhaled salbutamol) is the key issue in diagnosis of asthma.

Imaging studies such as plain chest X-ray (option B) or CT scan (option A) are not routinely used for diagnosis of asthma, unless bases on clinical grounds, an alternative diagnosis other than asthma is considered.

Spirometry (option C) can help in diagnosis by showing a reversible obstructive pattern; however, it is of little value during an exacerbation of asthma when patient cannot efficiently follow the instructions such as taking a very deep breath, or blowing with the maximum force they can. In patients with intermittent asthma, pulmonary function test using spirometry may be inconclusive if carried out in between attacks. If so, bronchoconstriction is induced by metacholine (metacholine challenge test). A bronchodilator is then administered to see whether this is reversible. Reversibility suggests asthma with high certainty.

PEF (option E) is a valuable tool to use for monitoring the response to treatment e.g. inhaled salbutamol. Increasing values read by PEF indicates appropriate response to treatment. The question though asks about the best method for diagnosis now in the acute setting the patient has presented with not how to assess the response to treatment in which case PEF would be the correct answer.

57
Q

Which one of the following is the most common finding in a patient with pleural effusion?

A. Cough.
B. Chest tightness.
C. Dyspnea.
D. Weight loss.
E. Pleuretic chest pain.

A

C. Dyspnea.

Dyspnea is the most common and prominent clinical finding in pleural effusion.

Option A - Cough may be present in patients with pleural effusion and is often mild and non- productive. More severe cough or bloody or purulent sputum associated with cough suggest an underlying pneumonia or endobronchial lesions.

Option B - Chest tightness is not a feature caused by pleural effusion.

Option D - Pleural effusion per se does not cause weight loss. Weight loss, however, can be caused by the underlying condition such as TB, lymphoma, etc.

Option E - pleuretic chest pain is present in some patients and suggests an exudative etiology, such as pleural infection, pulmonary infarction, or mesothelioma.

58
Q

A 35-year-old woman presents to the Emergency Department with sudden onset shortness of breath and right-sided pleuretic chest pain. On examination, she has a blood pressure of 130/89 mmHg, pulse rate of 102 bpm, and respiratory rate of 25 breaths per minute. Neck and forehead veins are flat and the trachea is slightly deviated to the right side. Her right-lung field has reduced breath sounds and is hyper-resonance to percussion. A chest X-ray shows a 25% right-sided pneumothorax. There are no other abnormalities on chest X-ray. Which one of the following is the most appropriate next step in management?

A. Chest tube drainage.
B. Needle aspiration.
C. Intubation and mechanical ventilation.
D. Observation for 24 hours.
E. Thoracotomy.

A

B. Needle aspiration.

This woman has developed pneumothorax in the absence of a history of trauma or an invasive procedure. Such pneumothoraces are termed ‘spontaneous pneumothorax’. She has no chest X-ray abnormalities apart from those related to the pneumothorax. She also has no history of lung disease, making ‘primary spontaneous pneumothorax (PSP)’ the most likely diagnosis. The management option of choice for patients with symptomatic PSPs or asymptomatic PSPs that are large (>15%) is needle aspiration.

Option A Chest tube insertion is the management of choice in patients with traumatic pneumothorax, patients with a large (>15%) secondary spontaneous pneumothorax, or when a patient with pneumothorax is about to undergo mechanical ventilation or being transferred by air (see topic review).

Option CMechanical ventilation or positive pressure ventilation of any kind makes pneumothorax worse.

Option D Observation is an acceptable management option for patients with primary spontaneous pneumothoraces that are asymptomatic and small (≤15%).

Option E Thoracotomy has no role in management of pneumothorax but may be indicated for the underlying airway disease as the cause of the pneumothorax.

TOPIC REVIEW

Pneumothorax is defined as presence of air in the pleural space. Based on the etiology, pneumothorax can be defined as traumatic or spontaneous.

Traumatic pneumothorax is caused by blunt or penetrating chest traumas or invasive procedures (iatrogenic) such as central vein catheterization, pleural biopsy, etc.

Spontaneous pneumothorax is the term used when the condition occurs in the absence of an apparent trauma or procedure. Spontaneous pneumothorax is classified as primary or secondary based on absence or presence of an underlying lung pathology or chest X-ray abnormalities.

-Primary spontaneous pneumothorax (PSP) – spontaneous pneumothorax is primary if all of following criteria are met:
* There is no respiratory finding on exam except those related to the pneumothorax
* There is no history of lung disease
* There is no chest X-ray finding other than those related to the pneumothorax
* The patient is young (≤50 years)
* There is no significant history of smoking

-Secondary spontaneous pneumothorax (SSP) – spontaneous pneumothorax is considered secondary if either of the following is present:
* Respiratory findings other than those related to pneumothorax
* History of lung disease such as COPD (the most common cause of SSP), asthma, cystic fibrosis sarcoidosis, TB, lung cancer, chronic bronchitis, bronchiectasis, extra- or intrathoracic restrictive lung disease, etc
* Any chest X-ray abnormality other than those related to pneumothorax
* Age >50 years
* Significant history of smoking

NOTE - the typical patient with PSP is a thin tall man of 20- to 40-years old. PSP is rarely seen after 40 year and almost never after the age of 50; therefore, spontaneous pneumothorax in those older than 50 is considered secondary and treated accordingly until proven otherwise.

Size of pneumothorax - Accurate estimation of the size of a pneumothorax is difficult. There different methods for estimation:

Average interpleural distance (AID) method - approximates the size of a pneumothorax from a PA CXR in standing position by taking the sum of the distances (measured in millimeters)
between the ribs and the visceral pleura at the apical level (A), midthoracic (B), and basal level (C), then dividing the sum by 3.

Light index - An upright PA chest X-ray is obtained. The width of the lung and the hemithorax are measured (in centimeters). The percentage of pneumothorax is calculated from the following formula:

Pneumothorax percentage = (1 - (width of the lung)^3 / (width of the hemithorax)^3) x 100

*For example if the width of the lung and the hemithorax are 5cm and 10cm respectively, the percentage of pneumothorax will be:

1 - (5)^3 / (10)^3 = 1 - 0.125 = 0.875 or 87.5% (87.5% of the affected hemithorax is occupied with the pneumothorax)

These methods are difficult to apply in practice and often underestimate the size of the pneumothorax. As a result, some clinicians tend to describe a pneumothorax as large or small, rather than using the percentage.

Chest wall - pleural line distance at the hilum level - British Thoracic Society guidelines define a pneumothorax as small if the distance from chest wall to the visceral pleural line (at the
level of the hilum) is less than 2 cm or large if the distance from the chest wall to the visceral pleural line is 2 cm or greater. Some clinicians prefer 3 cm laterally and 4 cm apically as the threshold to distinguish small and large pneumothoraces.

Symptoms:
* Small pneumothoraces are often asymptomatic.
* Larger pneumothoraces can present with:
-Pleuretic chest pain (pain may be referred to shoulder tip)
-Shortness of breath

Clinical findings associated with pneumothoraces include:

  • Decreased breath sounds over the affected area due to decreased air entry
  • Hyperresonance over the affected area
  • Decreased tactile fremitus
  • Tracheal deviation to the affected side

Management:

Primary spontaneous hemothorax (PSP) – management of PSP depends on the presence of the symptoms and/or the size of the pneumothorax:

Consider discharging the patient and review in 24 hours, and every 1-2 weeks until the pneumothorax is resolved if:
* The patient is asymptomatic (or minimal symptoms) AND
* The size of the pneumothorax is less than 15% of the affected lung, or the distance between the chest wall and the visceral plural line is ≤2cm

Consider needle aspiration and REVIEW if:
* The patient is symptomatic (pleuretic chest pain or dyspnoea), OR
* The size of the pneumothorax is ≥15%of the affected lung, or the distance between the chest wall and the visceral plural line >2cm

After needle aspiration, admission and catheter drainage (chest tube) would be indicated as the next best step in management if:
* The aspirated air is ≥3 litres
* The distance between the chest wall and visceral pleural line is still >2cm on a chest x- ray taken 4 hours after needle aspiration
* There is significant shortness of breath

NOTE - the rationale behind less invasive initial management of PSP is based upon the fact that the patients with PSP are young and otherwise healthy, and there is no underlying pathology to perpetuate the pneumothorax or prevent it from spontaneous healing

Secondary spontaneous pneumothorax (SSP) – in SSP, the underlying lung problem prevents the pneumothorax from spontaneous healing. All patients with SPS need to be admitted to the hospital and undergo:

  • Needle aspiration and REVIEW if there are no significant symptoms AND the size of the pneumothorax is less than 15% of the affected lung (or the distance between the chest wall and the visceral plural line is ≤2cm)
  • Catheter drainage (chest tube) if the patient is symptomatic (pleuretic chest pain or dyspnoea) OR the size of the pneumothorax is ≥15%of the affected lung (or the distance between the chest wall and the visceral plural line >2cm)

Traumatic pneumothorax

Unless the patient is asymptomatic and the size of pneumothorax is <15% (or the distance between the chest wall and the visceral plural line is ≤2cm), he patient should undergo catheter drainage (chest tubes). Asymptomatic patients whose pneumothorax is 15% (or the distance between the chest wall and the visceral plural line is ≤2cm) cab be closely observed for spontaneous resolution of the pneumothorax.

NOTE - pneumothorax of any kind and size should be treated with chest catheter (chest tube) insertions if:
* The patient is undergoing general anaesthesia for any reason
* The patient is planned to be intubated and mechanically ventilated
* The patient is planned to be transported by air (air transport)

59
Q

Which one of the following options is the most appropriate test to differentiate pulmonary tuberculosis (TB) from sarcoidosis?

A. Chest X-ray (CXR).
B. Erythrocyte sedimentation rate (ESR).
C. C-reactive protein (CRP).
D. Mantoux test.
E. Sputum exam.

A

D. Mantoux test.

Sarcoidosis and pulmonary TB have many similarities in presentation. They both can be associated with vague systemic symptoms of fever, night sweats, and weight loss. They both cause cough and might appear similar on CXR and other imaging modalities. There might be also an overlap syndrome in which both conditions coexist.

Both TB and sarcoidosis are granulomatous diseases; however, tuberculosis has a caseating granuloma as opposed to sarcoidosis, which presents with non-caseating epithelioid cell granuloma.

Misdiagnosis of sarcoidosis as TB results in futile anti-TB treatment while the sarcoidosis remains unattended. Conversely, a patient with misdiagnosed TB as sarcoidosis may be treated with corticosteroids that worsen the TB due to induction of immunodeficiency. Therefore, it is imperative to distinguish between these two. A tuberculin skin test (TST), also known as Mantoux test, is the most appropriate initial tool to apply for differentiation between these two. A negative TST suggests sarcoidosis versus TB. A positive test, however, should prompt extensive investigations for TB. A patient with a positive TST may be infected with TB, or the result might just be false positive e.g. due to previous BCG vaccination.

(Option A) CXR can be quite similar in both TB and sarcoidosis; hence, not a good means of differentiation.

(Options B and C) ESR and CRP are non-specific inflammatory markers that could be elevated in both sarcoidosis and TB and are not of discriminatory value.

(Option E) Sputum exam can be inconclusive in patients with TB, and smear-negative TB is frequently encountered. Therefore, negative sputum satin for AFB or culture cannot exclude TB as a diagnosis.

NOTE histopathology study on specimens from lung biopsy is the most accurate and gold standard method for distinguishing sarcoidosis from TB.

60
Q

A 30-year-old woman, recently migrated from China to Australia, is undergoing a health assessment. Before coming to Australia, she had been working in a cotton mill for 18 months and describes that upon start of the workweek she developed chest tightness, shortness of breath, cough, and wheeze. These symptoms eased up through the week, were almost absent in weekends, and started over the next workweek. Physical examination including a thorough examination of the chest and lungs are inconclusive. Her chest x-ray is normal as is her pulmonary function tests. Which one of the following could be the most likely diagnosis?

A. Anxiety.
B. Berylliosis.
C. Byssiniosis.
D. Silicosis.
E. Occupational asthma.

A

C. Byssiniosis.

The clinical picture given in this scenario is highly suggestive of byssinosis as the most likely diagnosis. Byssinosis is a type of occupational lung disease almost exclusively seen in workers who are in contact with unprocessed raw cotton especially those who are exposed to open bales of cotton or work in cotton spinning or carding rooms.

Often, byssinosis occurs in workers with at least 10 years of exposure, but it can also develop acutely in a matter of hours or days.

The etiology of the condition is suggested to be a bacterial endotoxin in cotton dust. The endotoxin results in bronchoconstriction acutely, and in longer term exposures, chronic bronchitis and gradual decreases in pulmonary function, particularly in genetically susceptible people. Cotton dust does not cause chronic obstructive lung disease as (COPD) as previously thought.

Symptoms are chest tightness and dyspnea that reduce with repeated exposure. Symptoms develop on the first day of work after a weekend or vacation and diminish or disappear by the end of the week. With repeated exposure over a period of years, chest tightness tends to return and persist through midweek and occasionally to the end of the week or as long as the person continues to work. This typical temporal pattern distinguishes byssinosis from asthma ; however, symptoms are relieved by asthma medications. Physical findings of acute exposure are tachypnea and wheezing while with chronic exposure crackles can be a finding.

During attacks, pulmonary function tests show characteristic patterns of reversible air flow obstruction and reduced vital capacity, especially if measured during the first work shift. Other than such times, pulmonary function tests are inconclusive unless chronic disease has set in, in which case a restrictive pattern is more likely. Hyperresponsiveness to methacholine is often observed.

(Option A) Anxiety-induced panic attacks may mimic parts of this clinical picture such as chest tightness and shortness of breath. If anxiety triggers an asthma attack in an asthmatic patient, the clinical picture can be identical to the scenario. However, with the exposure to cotton in history and the temporal course of symptoms in this patient (only present when she is exposed to the irritant) byssinosis is a better explanation and the more likely cause of this clinical picture.

(Option B) Exposure to and inhalation of beryllium (a metal used in aerospace, computer, electrical devices, and car industries) is associated with two pulmonary syndromes, i.e., (1) an acute chemical pneumonitis and (2) a granulomatous lung disease known as chronic beryllium disease (CBD), or berylliosis . In acute beryllium disease, the metal acts as a direct chemical irritant, causing a nonspecific inflammatory reaction (acute chemical pneumonitis). Berylliosis (chronic form) presents with shortness of breath, unexplained cough, fatigue, weight loss, fever, and night sweats. Some workers may develop severe symptoms very quickly, while others may not experience signs and symptoms until months or years after their exposure to beryllium. Berylliosis can continue to progress even after exposure has been removed. With no beryllium exposure in history, berylliosis is unlikely to be the cause of such presentation. Moreover, the clinical picture and temporal course of the symptoms in this patient are inconsistent with berylliosis.

(Option D) Silicosis is a progressive, irreversible, and incurable fibrotic pulmonary disease caused by the inhalation of respirable crystalline silica dust (dust diseases). There three types of silicosis:

  • Acute silicosis – Induced by short-term (<3 years) exposure to very high levels of silica. It presents with rapidly progressive dyspnea and respiratory failure with a high mortality rate.
  • Accelerated silicosis – Associated with exposure to high levels of silica over 3-10 years and presenting with clinical and radiological features of both acute and chronic silicosis. There is a higher rate of disease progression compared to chronic silicosis.
  • Chronic silicosis – occurs with exposures over 10 years. It can manifest as simple silicosis or complicated silicosis. Simple silicosis is often asymptomatic with small, predominately upper lobe nodules less than 1cm in size. It may progress to complicated silicosis. In complicated silicosis, (also known as progressive massive fibrosis (PMF)) nodules conglomerate into masses greater than 1cm in size. Calcification in masses and in hilar and mediastinal lymph nodes in common. As the condition progresses, lung function becomes impaired with development of dyspnea, and potentially cor pulmonale, respiratory failure and death.

The absence of exposure of to silica dust as well as the temporal course of the symptoms in this patient make silicosis unlikely.

61
Q

Peta, aged 3 years, is brought to the Emergency Department with an acute attack of asthma. She is initially treated with salbutamol inhaler through spacer and mask and oxygen by face mask. She improves symptomatically, but becomes breathless and wheezy again after 90 minutes. Which one of the following would be the most appropriate next step in management?

A. Double the dose of salbutamol.
B. Repeat salbutamol and commence oral corticosteroids.
C. Assess the severity of the asthma with spirometry.
D. Measure oxygen saturation to determine the need for hospitalization.
E. Start her on in intravenous aminophylline.

A

B. Repeat salbutamol and commence oral corticosteroids.

Peta initially responded to short-acting beta 2 agonists (salbutamol), but has deteriorated after 90 minutes. The best initial course of action would be repeating salbutamol to treat the current symptoms as well as early commencement of systemic corticosteroids to prevent further attacks. Every dose of short-acting beta 2 agonists is up to 12 puffs for those 6 years or older and up to 6 puffs for children 0-5 years. Exceeding the safe limits by doubling the dose (option A) is not appropriate.

Assessment of the severity of the attack, O2 saturation (option D) and the need for hospitalization are components of the management plan, but not the first priority. spirometry (option C) in the acute setting is not a reliable indicator of severity.

Intravenous aminophylline (option E) may be rarely indicated for patients with severe asthma attack and no or inadequate response to other measures.

62
Q

Ashley is brought to your clinic by his mother. He is a 9-year-old boy, who has been diagnosed with intermittent asthma two years ago. He is on inhaled salbutamol on an as-needed basis. His mother mentions that he has had to use his medication more frequently recently. Moreover, he has developed night coughs at least 2-3 times a week. Examination reveals diffuse wheezing over the lung fields. Which one of the following would be the most appropriate next step in management?

A. Start him on inhaled fluticasone and salmeterol.
B. Start him on inhaled fluticasone.
C. Start him on a 14-day course of oral steroids.
D. Start him on montelukast.
E. Start him on sodium cromoglycate.

A

D. Start him on montelukast.

This child had been diagnosed with mild intermittent asthma and started on a short-acting β2 agonist (salbutamol) on an as-needed basis, but his asthma is poorly controlled. Once adherence to therapy and inappropriate medication use are excluded as the possible causes of unresponsiveness to treatment, the next step would be a 2- to 4-week course of montelukast. If the symptoms improve, montelukast should be continued; otherwise, it should be replaced with the lowest effective dose of inhaled corticosteroids as the most appropriate next step in management. Montelukast is used to spare the child from inhaled corticosteroids and their potential adverse effects if there is a response.

63
Q

A 55-year-old man presents to your practice with complaints of shortness of breath and cough for the past 2 months. After initial evaluation, you arrange a pulmonary function test for him, the result of which is as follows:

  • FVC (forced vital capacity) = 60% (normal >80% of predicted)
  • FEV1 (Forced expiratory volume in 1 second) = 35% predicted (normal >80% of predicted)
  • FEV1/FVC = 0.58

Which one of the following is the most likely diagnosis?

A. Obstructive pulmonary disease.
B. Restrictive pulmonary disease.
C. Obesity.
D. Myasthenia gravis.
E. Concurrent obstructive and restrictive lung pathologies.

A

E. Concurrent obstructive and restrictive lung pathologies.

Forced vital capacity (FVC) is the amount of air that a patient can exhale in a single expiration after a deep inspiration, no matter how long it takes. FVC is diminished (less than 80% predicted) in restrictive pulmonary disease and severe obstructive diseases.

Forced expiratory volume in the first second (FEV1) reflects the airway flow; therefore it is expected to be within normal range in patients with restrictive pulmonary disease, as there is no obstruction against the flow. FEV1 is diminished in obstructive diseases.

The best indicator of obstructive vs. restrictive pulmonary disease is the FEV1/FVC ratio (normal 0.75 – 0.85). In obstructive diseases, the FEV1 is decreased, but the FVC remains unaffected or mildly decreased (unless there is severe obstruction); hence, the ratio will decrease. In restrictive diseases both FEV1 and FVC decrease so the ratio is expected to be normal or even slightly increased.

When the FEV1/FVC is reduced, the next step would be looking at the FVC. If FVC is normal a pure obstructive disease (e.g. asthma) is the expected underlying cause, whereas a decreased FVC is either due to a reduction in lung volume (e.g. concomitant restrictive lung disease), namely mixed pattern, or severe obstruction that can result in increased residual volume and consequently a decreased vital capacity.

Since this patient has decreased FEV1, FEV1/FVC, and FVC, a severe obstructive disease or a mix obstructive/restrictive lung pathology is the best explanation and the correct option.

64
Q

A 35-year-old man with history of sore throat three weeks ago, presents to the Emergency Department with a puffy face and decreased urine output for the past 48 hours. On examination, he has a blood pressure of 160/120 mmHg. His face is swollen especially in the periorbital area. On lung auscultation, bilbasal crepitations are heard. Which one of the following is correct regarding his condition?

A. Hematuria is a grave prognostic factor.
B. He is at immediate risk of death from left ventricular failure.
C. Increase in oral intake will result in diuresis.
D. If there is renal tenderness, a renal biopsy should be performed. E. Dialysis is contraindicated during the acute phase of the illness.

A

B. He is at immediate risk of death from left ventricular failure.

Facial edema, hypertension and oliguria are strong pointers towards glomerulonephritis. With the sore throat in the history, post-streptococcal glomerulonephritis (PSGN) is the most likely cause of this presentation.

PSGN is induced by infection with specific nephritogenic strains of group A streptococcus (GAS) such as type 12 and type 49. The clinical presentation can vary from asymptomatic, microscopic hematuria to full-blown acute nephritic syndrome, characterized by red to brown urine, proteinuria (which can reach the nephrotic range), edema, hypertension and acute renal failure.

A latent period always occurs between the streptococcal infection and the onset of signs and symptoms of acute glomerulonephritis. In general, the latent period is 1-2 weeks after a throat infection and 3-6 weeks after a skin infection. The onset of signs and symptoms at the same time as pharyngitis is points towards immunoglobulin A (IgA) nephropathy rather than PSGN.

Dark urine (brown-, tea-, or cola-colored) is often the first clinical manifestation of PSGN. Dark urine is caused by lysis of red blood cells that have penetrated the glomerular basement membrane and have passed into the tubular system.

Periorbital edema is typical. The onset of puffiness of the face or eyelids is sudden. It is usually prominent upon waking up and, if the patient is active, tends to subside toward the end of the day. In some cases, generalized edema and other features of circulatory congestion, such as dyspnea, may be present. Edema is the result of a defect in renal excretion of salt and water. The severity of edema is often disproportionate to the degree of renal impairment. Nonspecific symptoms of PSGN can include general malaise, weakness, and anorexia that are present in 50% of patients. Approximately 15% of patients complain of nausea and vomiting.

Early death is extremely rare in children (<1%) but is significantly more common in adults (25%). This is secondary to congestive heart failure and azotemia. Left ventricular failure (congestive heart failure) is more common in adults (43%) than in children (<5%); therefore, this patient is potentially at immediate risk of congestive heart failure and death due to volume overload.

Nephrotic-range proteinuria is also more common in adults (20%) than in children (4-10%). Approximately 83% of adults have azotemia compared to 24-40% of children.

(Option A) Hematuria is seen in most patients with PSGN as in all other forms of glomerulonephritis. It can range from microscopic to gross (cola-or tea-colored urine). Hematuria is not associated with poor prognosis.

(Option C) Oliguria is present in 10-50% of cases. The oliguria is often transient and diuresis occurs within 1-2 weeks. Increased water intake does not result in diuresis.

(Option D) Kidney biopsy is not required, unless for those patient in whom other glomerular disorders are considered due to deviation from the natural course of PSGN or lack of a history of streptococcal infection. Recognition of PSGN in this patient is clear from the history and the clinical findings. Renal tenderness is not an indication for renal biopsy.

(Option E) Patients with PSGN have variable reduction in renal function, and some patients require dialysis during the acute episode.